Open navigation menu
Close suggestions
Search
Search
en
Change Language
Upload
Sign in
Sign in
Download free for days
0 ratings
0% found this document useful (0 votes)
342 views
151 pages
Edexcel - FP1 PDF
Uploaded by
RS123
AI-enhanced title
Copyright
© © All Rights Reserved
We take content rights seriously. If you suspect this is your content,
claim it here
.
Available Formats
Download as PDF or read online on Scribd
Download
Save
Save Edexcel - FP1.pdf For Later
0%
0% found this document useful, undefined
0%
, undefined
Embed
Share
Print
Report
0 ratings
0% found this document useful (0 votes)
342 views
151 pages
Edexcel - FP1 PDF
Uploaded by
RS123
AI-enhanced title
Copyright
© © All Rights Reserved
We take content rights seriously. If you suspect this is your content,
claim it here
.
Available Formats
Download as PDF or read online on Scribd
Carousel Previous
Carousel Next
Download
Save
Save Edexcel - FP1.pdf For Later
0%
0% found this document useful, undefined
0%
, undefined
Embed
Share
Print
Report
Download now
Download
You are on page 1
/ 151
Search
Fullscreen
Ua dala) Mathematics 1 Edexcel AS and A-level Modular MathematicsPearson Education Limited, a company incorporated in England and Wales, having its registered office at Edinburgh Gate, Harlow, Essex, M20 2JE. Registered company number: 872828, ‘Text © Dave Wilkins, Keith Pledger, Lee Cope, Greg Attwood, Geoff Staley, Laurence Pateman, Bronwen Moran 2008 12111009 08 10987654321 British Library Catalogu ng in Publication Data is available from the British Library on request. ISBN 978 0.435519 230 Copyright notice Al rights reserved. No part of this publication may be reproduced in any form or by any means (including photocopying or storing it in any medium by electronic means and whether or not transiently or incidentally to some other use of this publication) without the written permission of the copyzight owner, except In accordance ‘with the provisions of the Copyright, Designs and Patents Act 1988 or under the terms of a licence issued by the Copyright Licensing Agency, Saffron House, 6-10 Kirby Street, London ECIN 81S (www.cla.co.uk). Applications for the copyright owner's written permission should be addressed to the publisher. Edited by Susan Gardner ‘Typeset by Tech-Set Ltd Mlustrated by Tech-Set Lea Cover design by Christopher Howson Picture research by Chrissie Martin Index by Indexing Specialists (UK) Cover photo/illustration © Edexcel Printed in the UK by Scotprint Acknowledgements The author and publisher would like to th reproduce photographs: I the following individuals and organisations for permission to Fotolia / Solar Wind Studios pi; Digital Vision p32; Alamy / David Cheshite p41; Moviestore Collection p72; Science Photo Library / Sheila Terry p107; Corbis / Hoge Norden / epa p12 Every effort has been made to contact copyright holders of material reproduced in this book. Any omissions will be rectified in subsequent printings if notice is given to the publishers.Contents About this book 1 Complex numbers 1.1 Real and imaginary numbers 1.2 Multiplying complex numbers and simplifying powers of i 1.3. The complex conjugate of a complex number 1.4 Representing complex numbers on an Argand diagram 1.5 Finding the value of r, the modulus of a complex number z, and the value of 6, the argument of z 1.6 The modulus-argument form of the complex number z 1.7 Solving problems involving complex numbers 1.8 Solving polynomial equations with real coefficients 2 Numerical solutions of equations 2.1 Solving equations of the form f(x) = 0 using interval bisection 2.2. Solving equations of the form f(x) = 0 using linear interpolation 2.3 Solving equations of the form f(x) = 0 using the Newton-Raphson process 3 Coordinate systems 3.1 Introduction to parametric equations 3.2 The general equation of a parabola 3.3. The equation for a rectangular hyperbola and finding tangents and normals Review Exercise 1 4 Matrix algebra 4.1 Finding the dimension of a matrix 4.2. Adding and subtracting matrices of the same dimension 4.3 Multiplying a matrix by a scalar (number) 4.4. Multiplying matrices together 4.5. Using matrices to describe linear transformations 4.6 Using matrices to represent rotations, reflections and enlargements. 4.7 Using matrix products to represent combinations of transformations 48 Finding the inverse of a 2 x 2 matrix where it exists 4.9 Using inverse matrices to reverse the effect of a linear transformation 4.10 Using the determinant of a matrix to determine the area scale factor of the transformation 4.11 Using matrices and their inverses to solve linear simultaneous equations 32 33 35 38 4 42 45 52 72 73 74 76 77 82 86 90 95, 99 101 103Series 5.1 The } notation 5.2 The formula for the sum of the first m natural numbers, Yor 5.3. Formulae for the sum of the squares of the first natural numbers, 772, and for the sum of the cubes of the first 1 natural numbers, 7r? 5.4 Using known formulae to sum more complex series Proof by mathematical induction, 6.1 Obtaining a proof for the summation of a series, using induction 6.2 Using proof by induction to prove that an expression is divisible by a certain integer 6.3. Using mathematical induction to produce a proof for the general terms of a recurrence relation 6.4 Using proof by induction to prove general statements involving matrix multiplication Review Exercise 2 Examination style paper 107 108 110 114 116 122 123 127 130 133 137 142About this book This book is designed to provide you with the best preparation possible for your Edexcel FP4 unit examination: Brief chapter overview and * This is Edexcel’s own course for the GCE specification ____ ‘links’ to underline the * Written by a senior examining team at Edexcel: the chair importance of mathematics: of examiners, chief examiners and principal examiners pare ae * The LiveText CD-ROM in the back of the book contains a Sees to even more resources to support you through the unit. a * Amatching FP4 revision guide is also available. Finding your way around the book Every few chapters, a review exercise helps you consolidate Detailed contents your learning list shows which parts of the FP1 specification are covered in each section Each section begins with a statement of what is covered in the section Past exami questions are (marked '€° Concise leaming points | Each section ends -by- i) with an exercise me aoe | =the questions are = they are model carefully graded solutions and so they increase include examiners | in difficulty and hints J gradually bring you up to standard Each chapter has a different Each chapter ends with At the end of the book there is colour scheme, to help you find | _ a mixed exercise and a an examination-style paper. the right chapter quickly summary of key points.After completing this chapter you should be able to: add, subtract, multiply and divide complex numbers find the medulus and argument of a complex number show complex numbers on an Argand diagram ‘solve equations that have complex roots. Complex numbers Although complex numbers may seem to have few direct links with real-world quantities, there are areas of application in which the idea of a complex number is extremely useful. For example, the strength of an electromagnetic field, which has both an electric and a magnetic component, can be described by using a complex number. Other areas in which the mathematics of complex numbers is a valuable tool include signal processing, fluid dynamics and quantum The Aurora Borealis (Northern Lights) are mechanics,1.1 You can use real and imaginary numbers. When solving a quadratic equation in Unit C1, you saw how the discriminant of the equation could be used to find out about the type of roots. For the equation ax® + bx + ¢ = 0, the discriminant is b? ~ 4ac. If b? ~ 4ac > 0, there are two different real roots. If b? — 4ac = 0, there are two equal real roots. If B® ~ 4ac < 0, there are no real roots. In the case b? ~ 4ac < 0, the problem is that you reach a situation where you need to find the square root of a negative number, which is not ‘real’ To solve this problem, another type of number called an ‘imaginary number’ is used. ‘The ‘imaginary number’ (1) is called i (or sometimes j in electrical engineering), and sums of real and imaginary numbers, such as 3 + 2i, are known as complex numbers. A complex number is written in the form a + bi. You can add and subtract complex numbers. avn Z An imaginary number is a number of the form bi, where b is a real number (b € [i). Write (36) in terms of i. C36) = (GX —1 = V36 CH = Gi Ea This can be written as 2iv7 or (2/7) to avoid confusion with 2V7i. Write \(—28) in terms of i. a VC28) = (28 X 1) = V2B CD = V4V7/CT) = 2V7i or 2iV7 or (2V7)i Solve the equation x? + 9 = 0. Note that just asx? = 9 has two roots +3 and BV) = *3i‘Complex numbers A complex number is a number of the form a + bi, where a € B and bE R. For the complex number a + bi, ais called the real part and bis called the imaginary part. The complete set of complex numbers is called C. Example EJ Solve the equation x? + 6x + 25 = 0. Method 1 (Completing the square) 2+ 6x = (x +3) w+ Gxt (+3) (+3) +16=0 Method 2 (Quadratic formula) 1 25) Ina complex number, the real part and the imaginary part cannot be combined to form a single term. You can add complex numbers by adding the real parts and adding the imaginary parts. You can subtract complex numbers by subtracting the real parts and subtracting the imaginary parts. Bene Simplify, giving your answer in the form a + bi, where a € Et and b ER. a (245i) + (7 + 3i) b B~ 4) +(-5 + 6H) © 2(5 ~ 8i) a (1+8i)- +i) e 2-5) -@- 11) £ (243) -@-3i)CHAPTER 1 a (2+5i) + (7+ 3) =(2+7) +15 +3)=9+ 81 ‘Add real parts and add imaginary parts. b -4) + (-5 +6) =(-5)+i(-44 6) =-2421 Ge) 4 (+6)-G6+ i) = 10 ~ 161 (1 6) +8 - 1) = -5 +7 This is the same as (6 - 8) + (5 - 8) Subtract real parts and subtract imaginary parts. e @-5i) —(6- ti) =(@-5) +i(-5 -(-) = 3 +61 f (2+3i) - (2-31 @-2 +16 Ca = 6 ‘The answer has no real part. This is called purely imaginary. ‘Simplify, giving your answer in the form @ + bi, where a € Mand b € R. 15 +21) + B+ 9) 3) (7 + Gi) + (-3 ~ Si) [$1 G-7i) + (-6 +71) 17) + 6i) ~ (8 + 101) 9 (-4- 6) - (8 — Bi) U1) G+ 4i) + (4+ Si) +5 + 61) 413) (18 + Si) - (15 - 2i) — B+ 71) 115, 38 — 4i) 17 28 +i) + 324i) 19 (2+ 3) + (G+ 9) Write in the form bi, where b € R. 21 v9) 23 (121) 25 \(-225) 27 12) 29 (200) 2 (4 + 10i) + 0 — Bi) | (@-i) + (1+ 2 (20 + 12i) — (11 + 3i) @-i)-C5+3i) (-1+5i)-(-1+9) (-2 - Ti) + (1 + 3i) = (-12 + i) 27 + 2i) 70 ~ 3i) 5(4 + 31) ~ 4(-1 + 2) Gv2 + i)- 2-1)‘Complex numbers Solve these equations. BI x? + 2x+5=0 32 x*-2x+10=0 33 x? + 4x + 29=0 34) x? + 10x + 26 =0 85) x? — 6x + 18=0 36 x2 +4x+7-=0 87 x2 - 6x + 11=0 38 x2 = 2x +25=0 89| x? + 5x + 25=0 40 x? + 3x+5=0 | 1,2 You can multiply complex numbers and simplify powers of You can multiply complex numbers using the same technique as you use for multiplying brackets in algebra, and you can simplify powers of i. M Since i= y(-1), 2 = -1 Multiply (2 + 3i) by (4 + Si) (2+ 2(4 + 5) = 2(4 + 51) + 34 +5) —___Malipty the two brackets as you a B +101 + 12i + 157? " Z = = 8+ 101+ 2-15 — _ SS = (6-15) + (01+ 12) ae ~ Add real parts and add imaginary parts. Express (7 ~ 4i)? in the form a + bi. NZ = it i’). Multiply the two brackets as you (7 = 4N(7 = 49) = 107 = 41) - 4107 - 41) a = 49 — 261 — 281 + 16? oon So Reee2e = Use the fact that i? ae | ‘Add real parts and add imaginaryCHAPTER 1 ‘Simplify (2 ~ 3i)(4 — $i) (1 + 3i) First multiply two of the brackets. ‘Then multiply the result by the third bracket. <7 ~ Bi — 22i — 66 = 59 — 431 © (2i)° Bali X i X 1X i X i) ——— First multiply the 2s (25). = Ba(i? x PX i) = 32 x -1X -1 x1 = 32i © (ANP = 2X 2X AX 2i x Zi Exercis ‘Simplify these, giving your answer in the form a + bi. 1) +i + 4) (2) 6+ 31017 + 2) 13 6-20 + 5i) 4 (13 ~ 32 ~ 81) (5) (-3- 4+ 7) 6) (8+ Si)? 7) (2-91)? 8 1+ )Z+)B +i) 19) B25 +:)4 - 21) 10 (2 + 3i)? ‘Simplify. 11 i6 12 (i)! 1 +i 14 (4i)' — 4" 15 (1 +i)* Hint: Use the binomial theorem.‘Complex numbers 1.3. You can find the complex conjugate of a complex number. I You can write down the complex conjugate of a complex number, and you can divide two complex numbers by using the complex conjugate of the denominator. The complex number a ~ bi is called the complex conjugate of the complex number a + bi. The complex numbers a + bi and a ~ bi are called a complex conjugate pair. The complex conjugate of zis called 7, so if 7= a + bi, z= a~ bi. Example {J} Write down the complex conjugate of a 243i bS—2i c 34+i ais Just change the sign of the imaginary part (from + to -, or = to +). Find z + z* and zz’, given that az=345i bz=2-7i © 2= 22 + W2 z+ 2 =(3+5i) + (B—5i) = (G+ 3) + iG — 5) = 6 -+— Notethatz + zis real. z= (3 + SiS — 5i) = 33 — Bi) + 5S — 5i) = 9 — i + 16i — 25% = 9 + 25 = 34 -—_________ Note that 27's real. bm a2tH _ zt2=(2-A+ (2+) =(2+2)+i-74+N=4 Note that z + z*is real. we = (2-72 + 7) = 22 + 7) — 72 + 7) ‘Note that zz’ is real. Note that z+ 2 is real. Note that zz’ is real.Simplify (10 + $i) + (1 + 2i) a a The complex conjugate of the (10 + 5i) + (1-4 2) = +51 x 1= 21, _1___ denominator is 1 ~ 2i. Multiply Tel ee numerator and denominator by this. 1o+ (lo + Sit - 21) T¥2i
a) ~ B) =x? ~ ax ~ Bx + ap =x? ~ (a+ B)x + of‘Complex numbers Example [J] Find the quadratic equation that has roots 3 + Si and 3 ~ Si. : a ‘This ny _For this equation a + f= (3 + Si) + (3 6 oe and afi = (3 + 5i)(3 — 5i) = 9 + 161 — 161 — 267 = 34 not required knowledge The equation is x* — Gx + 34 fo the REL eran, 1) Write down the complex conjugate z* for a 2-84 2i b z=6- ez=3-H dz=V5 + ivi0 2] Find z + 2 and zz* for az=6-3i b7=1045i ez=3+h dz=V5 - 3S Find these in the form a+ bi. 3) (25 ~ 101) + (I~ 2i) (4) ©+)+@+4i) 5| (11 +4i)+(3+i) @-4iP aes Given that z, = 1 + i, z) = 2+ iand zy = 3 + i, find answers for questions 12-14 in the form a + bi. (zy? 13 S+2i_ 15) Given that * 54 = 2 ~ j, find zn the forma + bi. giving your answer in the form a + bi. 16 Simplify $+ 8 t ss 17) The roots of the quadratic equation x? + 2x + 26 = O are a and f. Find a @andp batB © of18) The roots of the quadratic equation x? ~ &x + 25 = Oare «and B. Find aaandgp batBp ¢ of 19 Find the quadratic equation that has roots 2 + 3i and 2 ~ 3i, 20, Find the quadratic equation that has roots ~5 + 4i and ~5 ~ 4i. You can represent complex numbers on a diagram, called an Argand diagram. Areal number can be represented as a point on a straight line (a number line, which has one dimension). H Acomplex number, having two components (real and imaginary), can be represented as a point in a plane (two dimensions). The complex number z = x + iy is represented by the point (x,y), where.x andy are Cartesian coordinates. Ml The Cartesian coordinate diagram used to represent complex numbers is called an Argand diagram. The x-axis in the Argand Diagram is called the real axis and the y-axis is called the imaginary axis. The complex numbers z; = 2 + Si, z = 3 — 4i and z = — 4 + i are represented by the points A, Band C respectively on an Argand diagram. Sketch the Argand diagram. Imaginaryyt 4 @ 5) cca For 2, For z, For 23 = 2 + 5i, plot (2, 5). 3 4i, plot @, ~4). 41, plot 4, 1).‘Complex numbers Example [JJ Show the complex conjugates 2; = 4 + 2i and z’ = 4 ~ 2ion an Argand diagram. Imaginary» 4,2), a Note that complex conjugates will always be placed symmetrically above and below a z the real axis. Real 4-2), The complex number 7 = x + iy can also be represented by the vector OP; where 0 is the origin and Pis the point (x, y) on the Argand diagram. Show the complex numbers z, = 2 + Si, = 3 ~ 4iand z= ~4 + ion an Argand diagram, For z, = 2 + 5i, show the vector from (0, 0) to (2, 5). Similarly for zp and z3. If you label the diagram with letters A, B and C, make sure that you show which letter represents which vector.CHAPTER 1 ‘The complex numbers z; = 7 + 24i and z, = ~2 + 2i are represented by the vectorsOA and 0 respectively on an Argand diagram (where 0 is the origin). Draw the diagram and calculate |04| and {OB|. A (7,24) |CA| = VF +O = JER = 25 |0B| = \(-2)? + 2 = VB = 22 Addition of complex numbers can be represented on the Argand diagram by the addition of their respective vectors on the diagram. Seu 19} 4, = 4+ iand z, = 3 + 3i, Show z,, z, and z, + z, on an Argand diagram. a+ m= (443)+i(1+3)=7+ 41 Note that the vector for z1 + z (00 is the diagonal of the parallelogram. This is because OC = OA + AC- OA + OB.‘Complex numbers 4 = 6~ 2iand z, = 1 + 4i, Show 2), z and z; +z on an Argand diagram. 2+2,= (6-1) +i(-24+4)=5421 Note that the vector for 2, + 2, (00 is the diagonal of the parallelogram. This is because OC = OA + AC = OA + OB. 7 = 2+ Siand z, = 4 + 2i, Show z,, 7, and 7, ~ 7; on an Argand diagram. 4-H= H+ 2). The vector for -z, dotted line on the diagram.CHAPTER Cree L_ Show these numbers on an Argand diagram. a7+2i b 5-41 © -6-i d -2+5i e3 f 242i g h-4 2. Given that z, = -1~i, z= —S + 1iand z, = 3 ~4i, a find 247), 72, and 2 the forma + ib. D show 24, 7) Zs 212, Z1%3 and. z on an Argand diagram. 3) Show the roots of the equation x” — 6x + 10 = 0 on an Argand diagram, 4 The complex numbers 2; = § + 12i, z) = 6 + 10i, z = —4 + 2iand z, = -3 —iare represented by the vectors OA, OB, OC and OD respectively on an Argand diagram Draw the diagram and calculate |OA|, |OB|, |OC| and |OD). 5 72> 11 + 2i and z, = 2 + 4i, Show 2, and 2, + 2) on an Argand diagram, 16: 2 = ~3 + 6iand z, = 8 ~ i, Show z,, z, and z; + z onan Argand diagram. (7) 2 =8+ 4iand z = 6 + 7i. Show z,, z and z; — z on an Argand diagram. 8 z= ~6~ Si and z= ~4 + 4i, Show 4, z and z; ~ z on an Argand diagram. 1,5. You can find the value of r, the modulus of a complex number z, and the value of @, the argument of z. Consider the complex number 3 + 4i, represented ‘on an Argand diagram by the point A, or by the vector OA. The length OA or |OA|, the magnitude of vector |OAl, is found by Pythagoras’ theorem: |OA| = B+ 4? = 25 =5 This number is called the modulus of the complex number 3 + 4i.‘Complex numbers The modulus of the complex number z= x + iy is given by \x? + 9. The modulus of the complex number z = x + iy is written as r or |2| or |x + iy|, So r= yk? +9, |) = yar ey, [xe + ty| = yx? + 9? The modulus of any non-zero complex number is positive. Consider again the complex number z=344i, By convention, angles are measured from the positive x-axis (or the positive real axis), anticlockwise being positive. The angle @ shown on the Argand diagram, measured from the positive real axis, is found by trigonometry: tan o= 4, @ = arctan 4 = 0.927 radians This angle is called the argument of the complex number 3 + 4i. The argument of the complex number z = x + iy is the angle 0 between the positive real axis and the vector representing z on the Argand diagram. For the argument @ of the complex number z= x + iy, tan =. The argument 0 of any complex number is such that —7< 6< 7 (or —180° < @< 180°). (This is sometimes referred to as the principal argument). The argument of a complex number zis written as arg z. The argument 0 of a complex number is usually given in radians. Itis important to remember that the position of the complex number on the Argand diagram (the quadrant in which it appears) will determine whether its argument is positive or negative and whether its argument is acute or ‘obtuse, ‘The following examples illustrate this.‘CHAPTER 1 Find, to two decimal places, the modulus and argument (in radians) of z= 2 + 7i. Sketch the Argand diagram, showing ‘the position of ‘the number. Here zis in the first quadrant, so this angle is the required Modulus: |2| = [2 + 71] = v2? 4 7 = VBS = 2 dp. argument. fodulus: — |z| = |2 + 7i| = v2" VBS = 7.28 (2dp) eae Argument: tana =% a= 1.2924... radiane anticlockwise from the positive arg z= a radians (2 d.p.) real axis). Find, to two decimal places, the modulus and argument (in radians) of z = —5 + 2i. Sketch the Argand diagram, showing the position of the number. Here z is in the second quadrant, 50 the required a Modulus: [2 = |-5 + 2i| = \(-89) + @ = V2 = 5.39 (2 dp) fee creasured 2 =a = anticlockwise from Argument: tana = a a = 0.3805... radians ‘the positive real arg z = (m ~ 0.3808) = 2.76 radians (2 d.p) ak):‘Complex numbers Example §Z) Find, to two decimal places, the modulus and argument (in radians) of z= —4 ~ i. ‘Sketch the Argand diagram, showing ‘the position of the number. Here z is in the third quadrant, so the required argument is = 4.2 (2dp) ee fi 2 : (Clockwise from the GF = 02849... radians positive real axis is .90 radians (2 dp) negative). Modulus: |z] = |-4 ~ i] = y AP + ( Argument: tana = arg z = —(m — 0.2449) = — Find, to two decimal places, the modulus and argument (in radians) of z= 3 ~ 7i. Sketch the Argand diagram, showing ‘the position of "the number. Here zis in the fourth quadrant, - so the required =|3- 7] = \& + (-7) = VSB = 7.62 (2 ap) argument is 7 ~a (clockwise from Argument: tana =% a = 11659... radians the positive real axis is negative). 17 radians (2 dp.)Find the exact values of the modulus and argument (in radians) of z = Sketch the Argand diagram, showing the position of ‘the number. Here z is in the second quadrant, so the required = argument is a (ra) (measured anticlockwise from on ‘the positive real 4 axis). Modulus: |z| = |-1 + i] = y(-1)* +P = v2 Argument: tana arg z Find the modulus and argument of each of the following complex numbers, giving your answers exactly where possible, and to two decimal places otherwise. ii 12+ Si (2) v3 +i 3) -3+6i 4) 2-2 5 -8-7i Bu) 4+ 17) 23-3 (8) -8 15:‘Complex numbers 1.6 You can find the modulus—argument form of the complex number z. The modulus—argument form of the complex number z= + iy is Z= 1(cos 0 + i sin 6) where ris a positive real number and @/is an angle such that, =a < 0< 7 (or -180° < 0< 180°) From the right-angled triangle, X= ros @and y = rsin 6, ‘This is correct for a complex number in any of the Argand diagram quadrants. For complex numbers z, and z, |2,22| z1|l20)- Here is a proof of the above result. (You do not need to remember this proof for the exam!) Let |Z; ny, arg z, = 6 and |z;| = r, atg 2, = 6, 80 4 = n (cos & + isin &) and z = r(cos @ + isin &). (COS + isin 6) X r(cos & + isin 6) = Fyr{cos 6, + isin 6,)(cos & + isin 6.) iFA{COS 6, CoS &, — sin 6, sin 6, + i sin 6, Cos 6; + i Cos 6, sin é) ifal(CoS 6, COS 6; — sin @ sin 6) + i(sin 8, Cos é, + cos @ sin 6] Ake But (cos 6, cos @, — sin 6, sin 63) = cos (0, + 63) and (sin 6, cos & + cos 6, sin 6) = sin (0, + 6) So 22) = nypfcos (b + &) + isin (H + 6) You can see that this gives z\z, in modulus-argument form, with |z47)) = rf So |y22] = 11 72 = |eall2a) (Also, in fact, arg(2,z) = @ + 6)‘CHAPTER 1 a Express the numbers z; = 1 + iv and z, = ~3 ~ 3i in the form r(cos 6+ isin @). b Write down the value of |2:22|- Sketch the Argand diagram, showing the position of the numbers. {2s in the first quadrant, so this angle is the required argument (measured anticlockwise from the positive real axis). Modulus: Argument: Modulus: 2 isin the third quadrant, so the required argument is Argument: tan a = 8 a,=F Ponte hone cal axis is negative). So A= 2(co9 5 + sed z= BV (co0 (28) + 16in(-22)} Using |z,z,] Wife = 2 X BV2 = V2 = rite = [ailleel. [zee‘Complex numbers 1 we Express these in the form r(cos 6 + i sin @), giving exact values of rand @ where possible, or values to two decimal places otherwise. a2+2i b 3i © -3 + 4i 1-3 e -2-Si f —20 g 7-24 h-S+5i Express these in the form r(cos 6 + isin ®), giving exact values of rand @ where possible, or values to two decimal places otherwise. a3. 1+ 3 Write in the form a + ib, where a Rand b ER. a 32(cos F + isin) 4 4 b 6{cos 32 + i sin 32) ¢ vV3(cos $+ isin) a 7(cos(—J) + isin(—3)) © 4(cos(—52) + isin(-5)) In each case, find |24|, [zo] and 2,22, and verify that [242] = |z||Z2. an-3t4i 2-4-3. by=-1+2 2=442i 7+ 241 -v2 + Wd ©”_—-S+1a Zs dy=V3+N2 7, | 1.7 You can solve problems involving complex numbers. You can solve problems by equating real parts and imaginary parts from each side of an equation involving complex numbers. This technique can be used to find the square roots of a complex number. Hay + iy; =a + iy, thenx, =x, andy; = 92CHAPTER 1 Given that 3 + Si = (a+ ib)(1 + i), where a and b are real, find the value of a and the value of b. G@+ind+) = a+) ++) 1 =ataitbi-b =(a-b) + iat b) _S0 (a=W + (at H=345i i_a-b=3 Equate the real parts from each side of the ; ‘equation. Wo atp=5 Equate the i Addingi andi: 2a=6 of the equation. a=4 Substituting into equation i: ‘Solve equations i and ii simultaneously. Find the square roots of 3 + 4i. Suppose the equare root of 3 + Ais a+ ib, where a and b are real. Then (a + ib? =3 + 41 (a+ iba + ib) =3 4 4i ala + ib) + ia + ib) = 3 + 41 a@ + abi + abi- P = 3+ 4i (@ - P) + 2abi = 3 + 41 Equate the real parts from each side ‘of the equation. ____ Equate the imaginary parts from A each side of the equation. ~ 2a Substituting into a? — 4 = 3 Muttiply throughout by a. a4 = 38 iat — ee (#- 4a +1) =0 2 = Sora This is @ quadratic equation in o?.‘Complex numbers Since a is real, a* = —i has no solutions. Solutions are a = 2 or a = -2. Substituting back into b = 2: When a = 2, When a = —2, So the equare roots are 2 +i and —2 ~ i The square roots of 3 + 4i are (2 + ii) ret 1 aw 9 a + 2b + 2ai = 4 + 6i, where a and bare real, Find the value of a and the value of b. (ab) + (a+ Di =9 + Si, where a and bare real. Find the value of a and the value of b. (@+ DQ +i) = b+ 1+ (10 + 2a), where a and bare real. Find the value of a and the value of b. (a + i) = 18 + 261, where a is real. Find the value of a. abi = 3a — b + 12i, where a and b are real. Find the value of a and the value of b. Find the real numbers x and y, given that 7 gry 3 Find the real numbers x and y, given that @ +H) +)=24i Solve for real andy & +H) - 2) = -3 +71 Hence find the modulus and argument of x + iy. Find the square roots of 7 + 24i. 10 Find the square roots of 11 + 60. 11) Find the square roots of § ~ 121. 412, Find the square roots of 21.1,8 You can solve some types of polynomial equations with real coefficients. You know that, if the roots a and f of a quadratic equation are complex, a and B are always a complex conjugate pair. 1 Given one complex root of a quadratic equation, you can find the equation. I Complex roots of a polynomial equation with real coefficients occur in conjugate pairs. Example Ef) 7 + 2i is one of the roots of a quadratic equation with real coefficients. Find the equation. The roots are a conjugate The other root i 7 Bi $i pair. The equation with roots a and iis (x — a)(x— 3) =O ——7 — See page 8 and Example 14 (= (7+ 2) - (7 - 2) = 0 a x — 7 — 2i) — (7 + 2i) + (7 + 2i(7 — 21) =O a2 — Tx + Bix — 7x — Bix + 49 ~ 141 + 141 = 14x + 49+4=0 Si ella @— 14x + 53=0 An equation of the form ax? + bx? + ca + d= Ois called a cubic equation, and has three roots. Example Ei) Show that x = 2 is a solution of the cubic equation x* ~ 6x2 + 21x ~ 26 = 0. Hence solve the equation completely. For x = 2,x° — Gx + 2x — 26 = 8 - 24+ 42 - 26=0 : = 2 is a solution of the equation, so x — 2 is a factor 6x? + 2ix — 26 eae et x — 2h? — Gx? + Zix — 26 = 22 — 4x2 + 2ix Use long division (or : |______ another method) to find = 4x? + Bx the quadratic factor. 18x - 26 13x — 26 o‘Complex numbers 2 — 6x2 = 26 = (x - 2)(x2 - s 28 — Gx? + Bix — 26 = (x 2)(x® - 4x +13) = 0 + TSS Solving x2 — 4x + 13 = 0 y— found by the quadratic equation. Solve by completing the square. Alternatively, you could use the quadratic formula. ‘The quadratic equation has complex roots, which must be a conjugate pair. “X= 24 Bix = 2-31 S0 the 3 roots of Nn are 2,2 + Bi, and 2 — Bi Note that, for a cubic equation, either i all three roots are real, ‘or —_ii_ one root is real and the other two roots form a complex conjugate pair. Example Ed Given that ~1 is a root of the equation x ~ x2 + 3x + k= 0, a find the value of k, b find the other two roots of the equation. a_If-tie a root, 1? = 1? + 36 SS BE=0 5 b 1s. a root of the equation, 6o x + 1i¢ a factor of xt ihe —x*+3x+5 e+e — 2x? + Bx Use long division (or —+ ~ another method) to find the quadratic factor. x5 Sx +5CHAPTER 2-22 + Bx $5 = (x + IQ? - Be +5) =0 i ‘The other two roots are Solving x — 2x +5 =O —— found by solving the , re ee quadratic equation. w= x= NHI Solve by completing the x Ox +5 = (e—P 14S = (x1 +4 square. Alternatively, you pee ita (ia Lieitgat): could use the quadratic (e-1¥+4=0 a ‘The quadratic equation has complex roots, which must be a conjugate pair. 1+ 2x =1- 2i So the other two roots of the equation are | + Zi and ie An equation of the form ax‘ + bx? + cx? + da + e = Ois called a quartic equation, and has four roots. Given that 3 + i is a root of the quartic equation 2x ~ 3x? ~ 39x? + 120x ~ 50 = 0, solve the equation completely. Complex roots |= occur in conjugate pairs. Another root ie 3 x -G6x+9+1=0 x - 6x + 10=0‘Complex numbers So x* — 6x + 10 is a factor of 2x* — 3x° — 39x* + 120x — 50. (2 = Gx + 10)(ax? + bx + 0) 3x — 39x" + 120x — 50 EE _Coneider 2x* The only x4 term in the expansion lo x° X ai at ection term of the quartic separately. Consider __‘The x° terms in the expansion are x2 X bx and —Gx X 20 bx® — tax = —3x" b-12 sob=9, (x? — Gx + 10)(2x* + Ox + c) = 2x4 — Bx® — 39x" + 120x — 50 You can check this by considering the x and x? terms in. the expansion. ant term in the expan: = 39x? + 120x — nis 10 XG 606 Gx + 10)(2x? + Ox — aT Solving 2x + 9x —5 =O (x - (x +5) =0 So the roots of 2x* — 3x° — B9x* + 120x — 50 = O are 4,-6.3 +iand3 —i Note that, fora quartic equation, either i all four roots are real, or —_ii_ two roots are real and the other two roots form a complex conjugate pair, or —_ifi_ two roots form a complex conjugate pair and the other two roots also form a complex conjugate pair.‘CHAPTER 1 Example EZ) Show that x? + 4 isa factor of x4 — 2x3 + 21x? — 8x + 68. Hence solve the equation x* ~ 2x4 + 21x? ~ 8x + 68 = 0. Using long division: xe- 2x +17 2? + Ale = 239 + 2ix = Bx + 6B t+ 4st Itis also possible — 2x? + 17x? — Bx ani ae ex ‘considering each H+ 6B Seu all ae +68 Example 33. ° So xt — 2x° + 2ix? - Bx + 6B = (x? + 4)(x%- Be + 17) =O Either x + 4 = O or x*— 2x +17=0 Solving x2 + 4=0 a = x= 4 - x= t/CH = 24x) = evaVCD = 22 aaa Solving x* - 2x + 17= 0 Alternatively, you (= na could use the ae aed ‘quadratic formula. x= 2x 417 = (x= 1+ 17 = FH | @-1%+16=0 _ (x - 1)? = -16 x-1= +V(-16) = 241 x14 So the roote of x* — 2x? + 21 \_ 2, 21,1 + 4iand 1 - 41 erat jiven that 1 + 2i is one of the roots of a quadratic equation with real coefficients, find the equation. 8x + 6B = O are |2) Given that 3 — Si is one of the roots of a quadratic equation with real coefficients, find the equation. [3) Given that a + 4i, where a is real, is one of the roots of a quadratic equation with real coefficients, find the equation.‘Complex numbers 4) Show that x = -1 isa root of the equation x! + 9x? + 33x + 25 = 0. Hence solve the equation completely. 5 | Show that x ~ 3 is a root of the equation 2x' — 4x? — Sa — 3 Hence solve the equation completely. 6 | Show that x = —Lis a root of the equation 2x + 3x? + 3x + 1 = z Hence solve the equation completely. 7) Given that ~4 + iis one of the roots of the equation x* + 4x? ~ 18x ~ 68 = 0, solve the equation completely. 8) Given that x* ~ 12x3 + 31x? + 108x ~ 360 = (x? ~ 9)(x? + bx + 0), find the values of b and ¢, and hence find all the solutions of the equation x ~ 12x3 + 31a? + 108% — 360 = 0. 9) Given that 2 + 3i is one of the roots of the equation x! + 2x4 x? + 38x + 130 = 0, solve the equation completely. 40. Find the four roots of the equation x* ~ 16 = 0. Show these roots on an Argand diagram. 41, Three of the roots of the equation ax’ + bx* + ex? + de? + ex + f= Oare ~2, 2iand 1 +i Find the values of a, b, ¢, d, eand f. Mixed exercise [JJ 1 a Find the roots of the equation 2 + 2z + 17 = 0 giving your answers in the form a + ib, where a and b are integers. b Show these roots on an Argand diagram. oe 2 a Find the modulus of i 22 b Find the argument oft 27 Give your answers in radians as exact multiples of 7. oe _1 a)z=yh. a Express in the form a + bi, where a, b € B, iz itz-4. b Find (2%. ¢ Find arg(z — 2), giving your answer in degrees to one decimal place. o 4) The real and imaginary parts of the complex number z = x + iy satisfy the equation (2-ix- (1+ 3)y-7=0. a Find the value of x and the value of y. b Find the values of i (Z| ii arg z. oe‘CHAPTER 1 a Given that 2 + i is a root of the equation 24 ~ 11z + 20 = 0, find the other roots of the ‘equation. 16. Given that 1 + 3i is a root of the equation z* + 6z + 20 = 0, a find the other two roots of the equation, b show, on a single Argand diagram, the three points representing the roots of the equation, © prove that these three points are the vertices of a right-angled triangle. 17) 1-442) m= 341 a Display points representing z, and 7, on the same Argand diagram. bb Find the exact value of [z, ~ 2. Given that w= 7, © express win the form a + ib, where a, bE B, 4 find arg w, giving your answer in radians, 8) Given that 3 ~ 2i is a solution of the equation x! — 6x3 + 19x? — 36x + 78 = 0, a solve the equation completely, b show on a single Argand diagram the four points that represent the roots of the equation. a+3i 2+ai ‘a Given that a = 4, find |7|. aeR. b Show that there is only one value of a for which arg z = , and find this value.‘Complex numbers Summary of key points 1 JG =iand?? = -1. 2 An imaginary number is a number of the form bi, where b is a real number (b € R). 3 Acomplex number is a number of the form a + bi, where a € Rand b € R. 4 For the complex number a + bi, a is called the real part and b is called the imaginary part. 5 The complex number 2” = a ~ bi is called the complex conjugate of the complex number z=athi. 6 Ifthe roots a and f of a quadratic equation are complex, a and f will always be a complex conjugate pair. 7 The complex number z = x + iy is represented on an Argand diagram by the point (x, y), where x and y are Cartesian coordinates. 8 The complex number z = x + iy can also be represented by the vector OP, where O is the origin and Pis the point (x, y) on the Argand diagram, 9 Addition of complex numbers can be represented on the Argand diagram by the addition of their respective vectors on the diagram. 10 The modulus of the complex number z = x ~ iy is given by yx? + 9”. 11 The modulus of the complex number z = x ~ iy is written as ror [2] or[x + igl, so r= +y? |e] = ya? + be til = GP ty? 12 The modulus of any non-zero complex number is positive. 13 The argument arg z of the complex number z = x + iy is the angle @ between the positive real axis and the vector representing z on the Argand diagram. 14 For the argument @ of the complex number z = x + iy, tan @=%. 15 The argument 6 of any complex number is such that —7< 0< 7 (or —180° < 6 < 180°). (This is sometimes referred to as the principal argument.) 16 The modulus-argument form of the complex number z = x + iy is 2= (cos 6+ isin 0). [ris a positive real number and is an angle such that 7 <6< (or ~180° < 6 < 180°] 17 For complex numbers z; and 2,,|2,22| = |2i|22- 18 If.x, + iy = a) + Hye, then x, = x, andy, = yp 19 An equation of the form ax’ + bx? + cx + d = Ois called a cubic equation, and has three roots. 20 For a cubic equation, either a all three roots are real, or b one root is real and the other two roots form a complex conjugate pair. 21 An equation of the form ax* + bx* + cx? + dx + e = Ois called a quartic equation, and has four roots. 22 For a quartic equation, either a all four roots are real, or b two roots are real and the other two roots form a complex conjugate pair, or € two roots form a complex conjugate pair and the other two roots also form a complex conjugate pair.After completing this chapter you should be able to * find approximations to the solutions of equations of the form f() = 0 using: © interval bisection © linear interpolation © the Newton-Raphson process. Numerical solutions of equations - Numerical methods are used in science and engineering to help solve problems. These problems are normally modelled using computers. The numerical methods used lead to approximate solutions to the many equations that need to be solved. Weather forecasters use numerical methods to predict the weather, both in the immediate future (a few hours) and up to a few weeks ahead.the equation f(a) = 0. [Numerical solutions of equations Ifyou find an interval in which f(x) changes sign, then the interval must contain a root of You then take the mid-point as the first approximation and repeat this process until you get the required accuracy. Use interval bisection to find the pee Fommewmanns | root of VII to 1 decimal place. m as etn then x? = the so x -N=0 let f(x Al) = -10,4(2) = ~ (3) = —2, (4) = So the root lies between (3) and f(4) a F(a) b £(0) aye (224 | 5 |-2 4 |o 35 12 — a |-2 35 | 125 325 |-0.4975-—} 325 |-043% 35 | 125 325 |-04375 | 3.375 |0.3900625| 33125 | -0.0273437: 3.3125 | -0.0283437 | 3.375 | 0.390625 3375 | 0390625+y 334375| 0.180664 Hence vil = 33 to tap.fx) ‘The equation f(z) = 0 has a root x in the interval [2, 3]. or Using the end points of this interval find by interval bisection, a first and second approximation tox. Leta =2,b=3 at a fe) b fh 24 a (24 Make a table of values. 2 4 3 2 25 0.1669 2 25 01569 2.25 -0.493 " : eee) Use the change of sign 6 i =25 “tule to find the second et approximation = 25 nobilis 2nd approximation = 2.25 1 Use interval bisection to find the positive square root of x2 — place. 0, correct to one decimal 2 a Show that one root of the equation 2* ~ 7x + 2 = O lies in the interval [2, 3]. 'b Use interval bisection to find the root correct to two decimal places. 3. a Show that the largest positive root of the equation 0 = x' + 2x? — 8x — 3 lies in the interval [2, 3] b Use interval bisection to find this root correct to one decimal place. 4 a Show that the equation f() = 1 ~ 2sinx has one root which lies in the interval {0.5, 0.8). b Use interval bisection four times to find this root. Give your answer correct to one decimal place. %— 1.x > 0, has a root in the interval [1, 2]. b Obtain the root, using interval bisection three times. Give your answer to two significant figures. 5 a Show that the equation 0 = 6 f(x) = 6x — 3* ‘The equation f(x) = 0 has a root between x = 2 and x = 3. Starting with the interval [2, 3] use interval bisection three times to give an approximation to this root.‘Numerical solutions of equations In linear interpolation, you draw a sketch of the function f(x) for a given interval [a, b]. You then call the first approximation to the root of the function that lies in this interval x. ‘You use similar triangles to find a first approximation for x. You repeat the process using an interval involving the first approximation and one of the initial limits, where there is a change of sign to find a second approximation. Repeat until you find an approximation to the required degree of accuracy. a Show that the equation x° + 4x ~ 9 = chas a root in the interval [1, 2]. t 'b Use linear interpolation to find this root to one decimal place. a Let f(x) = x8 + 4x — 9 a Aha = 4 = f(2)=8+8-9=7 Since there ie a change of sign between f(1) and (2) the Rae equation x° + 4x — 9 = O has a root in the interval [1, 2]. b fea) 2-%_7 ee So B- 4x, = 7x,-7 15 = tx, x= B= 13686... al. #3626. 1.009767. Using the interval (1.36, 2) 2. £ —————7 f(1.4438607...) = -0.2144918... —~ Using the interval [1.443..., 2] r 2- Xs 7 x 144... O214.. Xs = 14603952 #(14603952) = -0.0437552 Using the interval [1.460..., 2] - Repeat the process again. Xq = 14637472, Hence root is 15 to one dip.‘Numerical solutions of equations f(a) = 3° — Sx The equation f(x) = 0 has a root « in this interval [2, 3]. ‘Using the end points of this interval find, by linear interpolation, an approximation tox. (2) = 1 (3) = 12 Draw a diagram for the interval. By cimmilar triangles 2 a Solve for x). x=8 15 to 2 dp. 1 a Show that a root of the equation x* ~ 3x ~ 5 = O lies in the interval [2, 3]. b Find this root using linear interpolation correct to one decimal place. 2 a Show that a root of the equation Sx* ~ 8x? + 1 = O has a root between x = 1 and x = 2. b Find this root using linear interpolation correct to one decimal place. 3. a Show that a root of the equation 3 + 3 = x lies in the interval [3, 4]. b Use linear interpolation to find this root correct to one decimal place. 4 a Show that a root of the equation 2x cosx — 1 = 0 lies in the interval [1, 1.5] 1b Find this root using linear interpolation correct to two decimal places. 5 a Show that the largest possible root of the equation 23 — 2x? — 3 = O lies in the interval [2, 3]. by Find this root correct to one decimal place using interval interpolation. 6 f@)=2-3r-1 ‘The equation f(x) = 0 has a root in the interval (3, 4]. Using this interval find an approximation tox.The Newton-Raphson formula is fGx,) Bnet =n Fey en) Note that the Newton-Raphson process may not always give you a better approximation and may take you further away from a root. Use the Newton-Raphson process to find the root of the equation x + x? = 80 which is near to / Xx = 3 correct to two decimal places Le fa) x4 Fx° 60 | EE Then (x) =4x9 + 2x let -%=5 =3 - 0.086 = 2.912 — Bae + 2a? — 80 The 25 — Se aie ae A(25R + 22912) = pain — 02656023 2912 ~ 107,.696058% 2908340184 So root = 291 to two decimal places. | Asbothay and maround to 291,‘Numerical solutions of equations f(x) = x9 + 2x? — Sx— 4 a Use differentiation to find f'(@). ‘The equation f(x) = 0 has a root a in the interval (1, 2] b Using 2 as a first approximation to a, use the Newton-Raphson process twice to find an. approximation for «. Give your answer correct to three decimal places. a f(x) = Bx? + 4x — 5 use hax = nxn + b Using xo = 2 ren x: i ———_ Use the Newton-Raphson process n=2-2 twice. aaa 6 x = 186 ao e a ‘Substitute x; = 1.86 into the ei) =~ RY n= ob - OBER — Xz = 18568 Xz = 1.856 to three decimal places 1 Show that the equation x3 ~ 2x ~ 1 = 0 has a root between 1 and 2. Find the root correct to ‘two decimal places using the Newton-Raphson process. 2 Use the Newton-Raphson process to find the positive root of the equation x! + 2x? — 6x ~ 3 = O correct to two decimal places. 3. Find the smallest positive root of the equation x* + x? ~ 80 = 0 correct to two decimal places. Use the Newton-Raphson process. 4 Apply the Newton-Raphson process to find the negative root of the equation x ~ Sx + 2-0 correct to two decimal places. 5 Show that the equation 2x* ~ 4x? — 1 = 0 has a root in the interval [2, 3]. Taking 3 as a first approximation to this root, use the Newton-Raphson process to find this root correct to two decimal places. 6 fa) = x8 - 3x2 + Se 4 ‘Taking 1.4 as a first approximation to a root, x, of this equation, use Newton-Raphson process ‘once to obtain a second approximation to x. Give your answer to three decimal places. 7 _ Use the Newton-Raphson process twice to find the root of the equation 2x* + 5x = 70 which is near tox = 3. Give your answer to three decimal places.8 Summary of key points 1 2 3 a 5 Given that f(x) = x* — 2x + 2 has a root in the interval [~1, ~2], use interval bisection on the interval [-1, ~2] to obtain the root correct to one decimal place. Show that the equation x’ ~ 12 ~ 7.2 = 0 has one positive and two negative roots. Obtain the positive root correct to three significant figures using the Newton-Raphson process. Find, correct to one decimal place, the real root of x* + 2v — Raphson process. = 0 by using the Newton Use the Newton-Raphson process to find the real root of the equation x* + 2x? + 4x ~ 6 = 0, taking x ~ 0.9 as the first approximation and carrying out one iteration, Use linear interpolation to find the positive root of the equation a¥ — Sx + 3 =0 correct to one decimal place. f(x) = x3 + x? — 6. a Show that the real root of f() = 0 lies in the interval [1, 2]. b Use the linear interpolation on the interval (1, 2] to find the first approximation tox. © Use the Newton-Raphson process on f(a) once, starting with your answer to b, to find another approximation tox, giving your answer correct to two decimal places. @ The equation cosx = x has a root in the interval [1.0, 1.4]. Use linear interpolation once in the interval [1.0, 1.4] to find an estimate of the root, giving your answer correct to two decimal places. f(a) = 25 — 3x6 Use the Newtor decimal places. Raphson process to find the positive root of this equation correct to two You can solve equations of the form f(x) = 0 using interval bisection. Ifyou find an interval in which f(x) changes sign, then the interval must contain a root of the equation f(x) = 0. You then take the mid-point as the first approximation and repeat this process until you get your required accuracy ‘You can solve equations of the form f(x) = 0 using linear interpolation. You can solve equations of the form f(x) = 0 using the Newton-Raphson process. ‘The Newton-Raphson formula is ‘The Newton-Raphson process may not always give you a better approximation and may take you further away from the root.Alter completing this chapter you should be able to: plot and sketch a curve expressed parametrically * work with the Cartesian equation and parametric equations of © aparabola © a rectangular hyperbola * understand the focus—directrix property of a parabola * find the equation of the tangent and the equation of a normal to a point © ona parabola © ona rectangular hyperbola. In this chapter you will be introduced to a parabola and its properties. You will also work with another curve called a rectangular hyperbola. This chapter builds upon the coordinate geometry work that you have learnt in units Core 1 and Core 2. For example, The shape of the cables found you will revise material on the distance, ais ornare gradient and mid-point between two ‘of the ways in which parabolas points (a, 94) and (xz, 92). . are used in the real world.chapreR 3 . | 3.1 You know what parametric equations are. Parametric equations are where the x and y coordinates of each point on a curve are ‘expressed in the form of an independent variable, say f, which is called a parameter. The parametric equation of a curve is written in the form: x= f(D, y = gO. You can define the coordinates of any point on a curve by using parametric equations. Sketch the curve given by the parametric equations x = at?, y = 2at, te where a is a positive constant. To give an idea of the shape of the curve we choose some values for t. Let's say t= -3,-2,-1,0,1,2.5. Sane | | em | [eld Ica |e) x=at?| 9a | 4a | a o a | 4a | 9a y= 2at|—-Ga|—4a|-2a| O | 2a | 4a | GaCoordinate systems ‘A curve has parametric equations x = at? y = 2at, £€ R where ais a positive constant. Find the Cartesian equation of the curve. cn) ee ales ee G0 x = Zee which simpitfies to Me -¥ 4a Hence, the Cartesian equation is ee A curve has parametric equations x = ct, y = & tER, t # O where cis a positive constant. a Find the Cartesian equation of the curve. b Hence sketch this curve. ae eran x=ct Se FS erage = efor, 7= E (2) : “e Hence, the Cartesian equation is &CHAPTER 3 1. A curve is given by the parametric equations x = 2t”, y = 4t. t€ IR. Copy and complete the following table and draw a graph of the curve for —4 << 4. t -4| -3 | -2 | -1 [-os] o [os] 1 | 2/3] 4 w= 20?| 32 0 | Os | 32 y=4t | -16 2 | 16 2) Acurve is given by the parametric equations x ~ 312, y = 6t. t€ R. Copy and complete the following table and draw a graph of the curve for ~3 = t= t -3 | -2 | -1 |-05] 0 0S 1 2 3 x = 3t? 0 ly =6t 0 3) Acurveis given by the parametric equations x = 4t, + 0. Copy and complete the following table and draw a graph of the curve for —4 = t -4| -3 | -2 | -1[-os[os] 1 [2] 3] 4 x= 4t | -16 2 y=4 | -1 “8 |Coordinate systems 4) Find the Cartesian equation of the curves given by these parametric equations. ax=5t,y = 10t ba=jty=t © x= 50t?, y = 100t dx=ley=2t e x=$t?,y=5t f x= v30,y = 23 gx =4ty = 20 hx = 6t,y=30 5) Find the Cartesian equation of the curves given by these parametric equations. 7 Z, 1 se! by bx=7t, t#0 © x= 3V5ty dx=hy= #0 6) Acurve has parametric equations x = 3t, y a Find the Cartesian equation of the curve. b Hence sketch this curve. mle TER, t #0. 7) Acurve has parametric equations x = v2t,y = “2, eR, t+ 0. a Find the Cartesian equation of the curve. b Hence sketch this curve. 3,2 You know the general equation of a parabola. The curve opposite is an example of a parabola which has parametric equations: x = at, y = 2at, te, where ais a positive constant. The Cartesian equation of this curve isy? = 4ax where a is a positive constant. This curve is symmetrical about the x-axis. I Ageneral point P on this curve has coordinates P(x, y) or P(at, 2at). ‘A ocus of points isa set of points which obey a certain rule. A parabola is the locus of points where every point P(@, y) on the parabola is the same stance from a fixed point 5, called the focus, and a fixed straight line called the directrix. The parabola isthe set of points where SP= PX. . The focus, 5, has coordinates (a, 0) Noy The directrix has equation x + a= 0. The vertex is at the point (0, 0). x+a=0‘CHAPTER 3 ea Find an equation of the parabola with a focus (7, 0) and directrix x +7 = 0 vB b focus (*8, 0) and directrix x = re a focus (7, 0) and directrix x+7=0 and directrix So parabola has equation y = vox. ee Find the coordinates of the focus and an ee for the directrix of a parabola with equation s je ee eee | aE the focus has coordinates (6, 0). and the directrix has equation x + 6 b = V52x- So the focus has coordinates (V2, 0). and the directrix has equation x + V2 = O.—y ee 4Coordinate systems Reminder “To find the distance d between two points (4, 14) and (2, yz) you can use the formula, d= en — mY + We ‘This formula can also be written in the form d= G2 — m+ Ye — Wl , A point P(x, y) obeys a rule such that the distance of P to the point (6, 0) is the same as the distance of P to the straight line x + 6 = 0. Prove that the locus of P has an equation of the form y” = 4ax, stating the value of the constant a, y ‘The (shortest) distance of P to the linex + 6 = 0 is the distance XP. x The distance SP is the same as the distance XP. The line XP is horizontal and has distance XP =x + 6. The locus of P is the curve shown. x+6=0 __ This means the distance SP is " From sketch the locus satisfies a8 the same as the distance XP. SP = XP Therefore, SF* = XF* _ Square both sides. So, (x = 6)F + (y - 0} L_ Used? = G2 — 1 + Wa - yo? i $564 —32+ 1x4 56 ft Tee C6). pa lenalayae ies Remember, (x ~ 6)? = (x — 6)(x ~ 6) which eirplifies to y# he locus of Phas an equation of “ine eerthotenn y= aa the form y? = 4ax where a a 50,a 03) 1 Find an equation of the parabola with a focus (5, 0) and directrix x + 5 = 0, b focus (8, 0) and directrix x + 8 = 0, © focus (1, 0) and directrix x = —1, focus (3, 0) and directrix x a a - © focus (4, 0) and cirectrixx +3 = 0, 2. Find the coordinates of the focus, and an equation for the directrix of a parabola with these equations. a y?= 120 by? = 20x cy? = 100 dy? = 43x ey? = vx f y?=5V2x 3 Apoint P(x, y) obeys a rule such that the distance of P to the point (3, 0) is the same as the distance of P to the straight line x + 3 = 0. Prove that the locus of P has an equation of the form y? = 4ar, stating the value of the constant a. 4) A point P(x, y) obeys a rule such that the distance of P to the point (2/5, 0) is the same as the distance of P to the straight line x ~ 2/5. Prove that the locus of P has an equation of the form y? = 4ax, stating the value of the constant a. A point P(x, y) obeys a rule such that the distance of P to the point (0, 2) is the same as the distance of P to the straight line y = -2. ‘@ Prove that the locus of P has an equation of the form y = kx”, stating the value of the constant k. Given that the locus of P is a parabola, b state the coordinates of the focus of P, and an equation of the directrix to P, © sketch the locus of P with its focus and its directrix. The point P(8, ~8) lies on the parabola C with equation y? = 8x. The point Sis the focus of the parabola. The line I passes through S and P. a Find the coordinates of S. b Find an equation for J, giving your answer in the form ax + by + ¢= 0, where a, b and care integers. The line / meets the parabola C again at the point Q. The point M is the mid-point of PQ. € Find the coordinates of Q. d Find the coordinates of M. € Draw a sketch showing parabola C, the line / and the points P, Q, and M.a y= 6x The focus, $ has coordinates (2, 0) + 2 I k By = 4x + 3y-8 The line |has equation 4x + 3y- 8 =0. 6 & 4x+3y-8=0 ()- 5 6x (2)-—_+ y¥ Bx + Gy - 16 =0 @)-—F i y=-8.2.——_ y+ 6y-16=0 (y + By — 2)= Or ‘The point Q has coordinates (4, 2). bth 4 The mid-point ie | 2, =8+2) 2 The point M has coordinates R mo Simplify Coordinate systems This is in the form y’ So 4a = 8, gives a= $= 2. Focus has coordinates (a, 0). oY ere (ry 11) = (2, 0) and Gy) — &, -8)- Use m= Use y — yr = m(x — x4). Here m = Gv ¥) = 2, 0). Multiply both sides by 3. Multiply out brackets. Simplify into the form ax + by + ¢= 0. ‘As the line I meets the curve C, we solve these equations simultaneously. Multiply (1) by 2. Substitute (2) into (3). Factorise. y= ~Bisat Pandy = 2isatQ Use y? = 8x and y = 2 to find the x-coordinate of Q Xt, Wit Ye a Z—} where P=, ys) = (8, ~8) and Q= G2.) = G2charTeR 3 1 The parabola Chae equation ¥ = 8x The line | has equation 4x + 3y-8=0. The line I cute the parabola at the points F(8, -8) and Q(%, 2). The points 5(2, 0) and M(1Z, —3) also lie on the line |. ‘The line y = 2x ~ 3 meets the parabola y? = 3x at the points P and Q. Find the coordinates of P and Q. ‘The line y = x + 6 meets the parabola y* = 32x at the points A and B. Find the exact length AB giving your answer as a surd in its simplest form. The line y = x — 20 meets the parabola y = 10x at the points A and B. Find the coordinates of A and B, The mid-point of AB is the point M. Find the coordinates of M. ‘The parabola C has parametric equations x = 6t?, y = 12t. The focus to Cis at the point S. ‘a Find a Cartesian equation of C. b State the coordinates of S and the equation of the directrix to C. © Sketch the graph of C. ‘The points P and Q on the parabola are both at a distance 9 units away from the directrix of the parabola, d State the distance PS. Find the exact length PQ, giving your answer as a surd in its simplest form. £ Find the area of the triangle PQS, giving your answer in the form kv2, where k is an integer. ‘The parabola C has equation y? = 4ar, where « is a constant. The point (5¢2, $1) is a general point on C. ‘a Find a Cartesian equation of C. The point P lies on C with y-coordinate 5. b Find the x-coordinate of P. The point Q lies on the directrix of C where y = 3. The line / passes through the points P and Q. © Find the coordinates of Q. Find an equation for J, giving your answer in the form ax + by + c= 0, where a, band ¢ are integers.Coordinate systems 6) A parabola C has equation y? = 4x. The point Sis the focus to C. a Find the coordinates of S. The point P with y-coordinate 4 lies on C. b Find the x-coordinate of P. ‘The line / passes through $ and P. ¢ Find an equation for /, giving your answer in the form ax + by + ¢ = 0, where a, b and ¢ are integers. ‘The line I meets C again at the point Q. d Find the coordinates of Q. € Find the distance of the directrix of C to the point Q. 7, The diagram shows the point P which lies on the parabola C with equation»? = 12x. ‘The point $ is the focus of C. The points Q and R lie on the directrix to C. The line segment QPis parallel to the line segment RS as shown in the diagram. The distance of PS is 12 units. a Find the coordinates of R and S. b Hence find the exact coordinates of P and Q. © Find the area of the quadrilateral PORS, gi integer. \g your answer in the form kv/3, where k is an 8! The points P(16, 8) and Q(4, b), where b < 0 lie on the parabola C with equation y? = 4az. a Find the values of a and b. Pand Qalso lie on the line . The mid-point of PQis the point R. b Find an equation of f, giving your answer in the form y = mx + ¢, where m and c are constants to be determined. © Find the coordinates of R. The line n is perpendicular to / and passes through R. i Find an equation of n, giving your answer in the form y = mx + c where m and c are constants to be determined. ‘The line n meets the parabola C at two points. € Show that the x-coordinates of these two points can be written in the form a = A + jw/I3, where A and x: are integers to be determined,ChareR 3. z 3,3. You know an equation for a rectangular hyperbola and can find tangents and normals. The curve opposite is an example of a rectangular hyperbola which has parametric equations: ' R= ay =F, tER, t#O where cis a positive constant. PO, 9) I The Cartesian equation of this curve is xy = ¢?, where cis a positive constant. The curve has asymptotes with equations x = 0 (the y-axis) and y = 0 (the x-axis), MA general point P on this curve has coordinates P(x, y) or (ct, £). ‘The point P, where x = 2, lies on the rectangular hyperbola H with equation xy = 8. Find a the equation of the tangent T, 'b the equation of the normal N, to Hat the point P, giving your answers in the form ax + by + ¢ = 0, where a, b and care integers. st When x =2,y=S=40— Ty -4=-2@- 2) Tax+y-B8=0 Therefore, the equation of the tangent to H at Pis ax +y-8=0.Coordinate systems b Gradient of tangent. at P is Che So gradient of normal is my = 4. Ney 4=3(-2)-——____ N2y-8=x-2 wer=nyt6=0n Therefore, the equation of ‘the normal to H at P is: x- 2y+6=0. The distinct points A and B, where x = 3 lie on the parabola C with equation ® = 27x. The line is the tangent to C at A and the line J, is the tangent to C at B. Given that at A, y > 0, a find the coordinates of A and B. b Draw a sketch showing the parabola C. Indicate on your sketch the points A and B and the lines J, and by. © Find: ian equation for h, i an equation for /, giving your answers in the form ar + by + ¢ = 0, where a, b and c are integers. a Gf =27% ¥ = 23) = 81 +VBI = £9-——— y Hence, the coordinates of A and B are (3, 9) and (8, —9) respective‘CHAPTER 3 The tangent line |, has a positive gradient. Hence, at A, the gradient of the re iti curve $ io pocitive. ‘The equation of the curve for y > O is y = +V27x! or y = +3V8x". The equation of the curve for y <0 is y = —V27x or y = —BV3x. The tangent line has a negative gradient. Hence, ab 6, the gradient of the ay F curve is negative. ax G y=2x y= VOR = AVENE = ET G0 y= +3V3x! Y _ 45/5 (1) 3 = +884 dx 7 TOVB(p)x* = 295° x 0, Y = 4 BVB O ax VE i AvAy> Oand 0 433 avx At Ax = 3 and =H v5 3 ea ae A has coordinates (3, ee ae Tt y-9=3(-3) Ti 2y — 18 = 3(x — 3) ye Matty oth ies by 2.) me Td TOx=2yFO= On to Cat Aic Bx ~ 2y +9 = 0.Coordinate systems li AtB,y
Oand eo When x = 3, my =compre 3 J ren 1 Find the equation of the tangent to the curve ay? = Axat the point (16, 8) b y® = 8x at the point (4, 4v2) © xy = 25 at the point (5, 5) xy = 4 at the point where x = > y= 7xat the point (7, —7) f xy = 16 at the point where x Give your answers in the form ax + by + ¢= 0. Find the equation of the normal to the curve a y= 20r at the point where y = 10, b xy = at the point (~3, -6). Give your answers in the form ax + by + ¢= 0, where a, b and care integers. The point P(4, 8) lies on the parabola with equation y? = 4ax. Find a the value of a, b an equation of the normal to C at P. ‘The normal to C at P cuts the parabola again at the point Q. Find the coordinates of Q, d the length PQ, giving your answer as a simplified surd, ‘The point A(—2, ~16) lies on the rectangular hyperbola H with equation xy = 32. a Find an equation of the normal to Hat A. ‘The normal to H at A meets H again at the point B. b Find the coordinates of B. ‘The points P(4, 12) and Q(-8, —6) lie on the rectangular hyperbola H with equation xy = 48. ‘a Show that an equation of the line PQis 3x — 2y + 12 = 0. ‘The point A lies on H. The normal to H at A is parallel to the chord PQ. b Find the exact coordinates of the two possible positions of A. ‘The curve H is defined by the equations x = V3t, y 5, tER, #0. ‘The point P lies on H with x-coordinate 2/3. Find: a a Cartesian equation for the curve H, b an equation of the normal to H at P. ‘The normal to H at P meets H again at the point Q. € Find the exact coordinates of Q The point P(4®2, 81) lies on the parabola C with equation y* = 16x. The point P also lies on. the rectangular hyperbola H with equation xy = 4. a Find the value of t, and hence find the coordinates of P. The normal to H at P meets the x-axis at the point N. b Find the coordinates of N. ‘The tangent to C at P meets the x-axis at the point T. © Find the coordinates of T. Hence, find the area of the triangle NPT.Coordinate systems ‘The point P(at?, 2at), lies on the parabola C with equation y? = 4ax where a is a positive constant. Show that an equation of the normal to C at P is y + te = 2at + at*. G y = 4ax y = V4ax = 2avx = 2a So y= 2vax ai ae = EE) eo So ee At P,x = at? and wi f-| aaa Tax Jat Vat t Gradient of tangent at Pie m, = 4. amen li — P has coordinates (at®, 2at). Ney + ta = 2at + at® Therefore, the equation of the normal to Cat Pis iad Sa The point P(ct, a t+ 0, lies on the rectangular hyperbola H with equation xy = @ where cis a positive constant. a Show that an equation of the tangent to Hat Pisx + Py = 2ct. ‘A rectangular hyperbola G has equation xy = 9. The tangent to G at the point A and the tangent to Gat the point B meet at the point (-1, 7). b Find the coordinates of A and B.CHAPTER 3 (ct? Gradient of tangent at Pie t y-£--le-) T: By - cb = -(e - ct) — T: Py — eb = -x + ct —____4j — Te xt Py = 2ct—___ Therefore, the equation of the tangent. to Hat Pie x + ty = 2ot. Compare G: xy = 9 with Bee as So, a6 cis positive, c = 3. Tangent to Giex + ty = Gt (1), — “1+ 1) = 6b —— 7? -6t-1=0 a (72+ Ne-1)=0 -1 taht P has coordinates 7. as. (§) = (8. 3)-—_ > When t = —4, the coordinates are a = {-3,— (4-)-2)-(4-2} When t = 1, the coordinates are 2\_ 3(1), 5} = (3.3). (20. @) = 9. Therefore, the coordinates 21) and (3, 3). 7 7, of A and Bare (—3, Rearrange the equation for Hin the form y = x". Differentiate to determine the gradient of H. Substitute x = cf, to calculate the gradient of the tangent to H. Use y ~ yi = my @e ~ ay) to find the equation of the tangent, T. Vand (ay, yy Here m, (ct, $). Multiply both sides by &. Multiply out brackets. Rearrange into the required form. @=95c= v9 sc=3, Substitute ¢ = 3 into the equation of the tangent derived in a. Substitute x = ~Tand y = 7 in (1) as tangent goes through point (—1, 7). Rearrange into a ‘quadratic equation = 0." Factorise. Solve. Substitute ¢ = 3 into the general coordinates of P. Substitute ¢ = Substitute t = 1 into P.Coordinate systems ren 1. the point P(3®, 6f) lies on the parabola C with equation y? = 12x. a Show that an equation of the tangent to C at Pis yt = x + 3°. b Show that an equation of the normal to C at Pis xt + y = 38+ 61. 2| ‘The point P{6t, ©), ¢ + 0, lies on the rectangular hyperbola H with equation ay = 36. a Show that an equation of the tangent to Hat Pisx + Py = 12t. b Show that an equation of the normal to IT at P is fx ~ ty = 6(t* ~ 1). 3 The point P(E, 101) lies on the parabola C with equation y? = dar, where a is a constant and 140. nd the value of a. a b Show that an equation of the tangent to C at Pis yt = x + SP. ‘The tangent to C at P cuts the x-axis at the point X and the y-axis at the point Y. The point 0 is the origin of the coordinate system. ¢ Find, in terms of f, the area of the triangle OXY. 4) The point Pat’, 2at), t + 0, lies on the parabola C with equation y? = 4ar, where ais a positive constant. a Show that an equation of the tangent to C at P is ty ‘The tangent to C at the point A and the tangent to C at the point B meet coordinates (—4a, 3a). b Find, in terms of a, the coordinates of A and the coordinates of B. + at, it the point with ‘The point P(4t, 4), t # 0, lies on the rectangular hyperbola H with equation ay = 16. a Show that an equation of the tangent to C at Pisx + Py = 8. ‘The tangent to Hat the point A and the tangent to H at the point B meet at the point X with y-coordinate 5, X lies on the directrix of the parabola C with equation y* = 16c. b Write down the coordinates of X. € Find the coordinates of A and B. _Deduce the equations of the tangents to H which pass through X. Give your answers in the form ax + by + ¢ = 0, where a, b and c are integers. 6 ‘The point Pat, 2at) lies on the parabola C with equation y? = 4ax, where a is a constant and £4 0, The tangent to C at P cuts the x-axis at the point A. ‘a Find, in terms of a and ¢, the coordinates of A. ‘The normal to C at P cuts the x-axis at the point B. b Fin, in terms of a and ¢, the coordinates of B. © Hence find, in terms of a and t, the area of the triangle APB. 7° The point P(2#, 40) lies on the parabola C with equation y? = &r. a Show that an equation of the normal to C at Pisxt + y = 20 + at. ‘The normals to C at the points R, S and T mect at the point (12, 0).. b Find the coordinates of R, S and T. © Deduce the equations of the normals to C which all pass through the point (12, 0).chaoTeR 3 ai 8) The point P(at, 2at) lies on the parabola C with equation y? = 4ax, where a is a positive constant and t # 0. The tangent to C at P meets the y-axis at Q. a Find in terms of a and ¢, the coordinates of Q. ‘The point $ is the focus of the parabola, b State the coordinates of S. © Show that PQis perpendicular to SQ. 9 The point P(6t, 124) lies on the parabola C with equation y? = 24x. SS a Show that an equation of the tangent to the parabola at P is ty =x + 6%, .. The point X has y-coordinate 9 and lies on the directrix of C. ~ B State the x-coordinate of X. | The tangent at the point B on C goes through point X. © Find the possible coordinates of B. 1 A parabola C has equation y* = 12x. The point $ is the focus of C. a Find the coordinates of S. ‘The line J with equation y = 3x intersects C at the point P where y > 0. b Find the coordinates of P. € Find the area of the triangle OPS, where O is the origin. 2, A parabola C has equation y? = 24x. The point P with coordinates (k, 6), where k isa constant lies on C. a Find the value of k. ‘The point $ is the focus of C. b Find the coordinates of S. ‘The line | passes through $ and P and intersects the directrix of C at the point D. © Show that an equation for lis 4x + ay ~ 24 = 0. d Find the area of the triangle OPD, where O is the origin. 3 The parabola C has parametric equations « = 122, y = 24f. The focus to Cis at the point S. a Find a Cartesian equation of C. The point P lies on C where y > 0. P is 28 units from S. b Find an equation of the directrix of C. € Find the exact coordinates of the point P. Find the area of the triangle OSP, giving your answer in the form k/3, where k is an integer. 4° The point (4t?, 82) lies on the parabola C with equation y? = 16x. The line I with equation 4x — 9y + 32 = 0 intersects the curve at the points P and Q. a Find the coordinates of P and Q b Show that an equation of the normal to C at (42, 81) isxt + y = 403 + 8t. © Hence, find an equation of the normal to C at P and an equation of the normal to C at Q. ‘The normal to C at P and the normal to C at Q meet at the point R. d Find the coordinates of R and show that R lies on C. Find the distance OR, giving your answer in the form ky97, where k is an integer.10 Coordinate systems The point P (at®, 2at) lies on the parabola C with equation y2 = 4ax, where a is a positive constant. ‘The point Q lies on the directrix of C. The point Qalso lies on the x-axis. a State the coordinates of the focus of C and the coordinates of Q. ‘The tangent to C at P passes through the point Q. b Find, in terms of a, the two sets of possible coordinates of P. ‘The point P(ct, fy ¢> 0, £4 0, lies on the rectangular hyperbola H with equation xy = a Show that the equation of the normal to H at Pis tx — ty = e(t* — 1). b Hence, find the equation of the normal rt to the curve V with the equation xy = 36 at the point (12, 3). Give your answer in the form ax + by = d, where a, b and d are integers. The line n meets V again at the point Q. € Find the coordinates of Q. A rectangular hyperbola H has equation xy = 9. The lines fy and /, are tangents to H. The gradients of I, and , are both ~1. Find the equations of , and Ip. The point P lies on the rectangular hyperbola xy ~ c2, where c > 0, The tangent to the rectangular hyperbola at the point P(ct, §) t> 0, cuts the x-axis at the point X and cuts the y-axis at the point Y. a Find, in terms of c and #, the coordinates of X and ¥. b Given that the area of the triangle OXY is 144, find the exact value of c. ‘The points P(4at?, 4af) and Q(16at”, 8at) lie on the parabola C with equation y? = 4ax, where ais a positive constant. a Show that an equation of the tangent to C at P is 2ty = x + 4at?. b Hence, write down the equation of the tangent to Cat Q. ‘The tangent to C at P meets the tangent to C at Qat the point R. € Find, in terms of a and f, the coordinates of R. A rectangular hyperbola H has Carte: t #0, t> Qis a general point on H. a Show that an equation an equation of the tangent to Hat (ct, § ) isx + Py = et. ‘The point P lies on H. ‘The tangent to H at P cuts the x-axis at the point X with coordinates (2a, 0), where ais a constant. in equation xy = ©, ¢> 0. The point (ct, 4) where 2 b Use the answer to part a to show that P has coordinates (a, & ‘The point Q, which lies on H, has x-coordinate 2a. ¢ Find the y-coordinate of Q Hence, find the equation of the line OQ where O is the origin. ‘The lines OQ and XP meet at point R. € Find, in terms of a, the x-coordinate of R. Given that the line OQ is perpendicular to the line XP, f Show that 2 = 2a, g find, in terms of a, the y-coordinate of R.charrer 3 Summary of key points 1 Tofind the Cartesian equation of a curve given parametrically you eliminate the parameter t between the parametric equations. 2 A parabola is a set of points which are y ‘equidistant from the focus S and a line | | called the directrix. Si So, for the parabola opposite, © SP= PX. ‘the focus, 5, has coordinates (a, 0). «the directrix has equation xta=0. —s directrix xt+a=0 3 The curve opposite is a sketch of a parabola with a Cartesian equation of y? = 4ar, where isa positive constant. This curve has parametric equations: x = af, y = 2at, teR, where ais a positive constant. 4 The curve opposite is a sketch of a rectangular hyperbola with a Cartesian. os ‘equation of xy = ©, where cis a positive constant. This curve has parametric equations: x= d,y=§,tER, t#0, where cis a positive constant. aacReview Exercise © = 2+i,4=3 + 4i Find the modulus and the tangent of the argument of each of 4 Lae e 243i a Show that the complex number 2-3! can be expressed in the form A(I+ i), stating the value of A. any 2+ 3i)* b Hence show that Sti determine its value. 22S +im=-2+3i a Show that Ij)? = 2lz,0. b Find arg@7,). a Find, in the form p + ig where p and q are real, the complex number z which satisfies the equation 32—! 'b Show on a single Argand diagram the points which represent z and 2’. © Express z and 7‘ in modulus~argument form, giving the arguments to the nearest degree. m= -1+iN3,2= v3 +i atind iamgz ii argz. bb Express Zin the form a + ib, where a and b are real, and hence find arg (71). Verify that arg (7!) = arg 2, — arg z. @ a Find the two square roots of 3 ~ 4i in the form a + ib, where a and b are real. b Show the points representing the two square roots of 3 ~ 4i in a single Argand diagram. ‘The complex number z is ~9 + 17i. a Show z on an Argand diagram. b Calculate arg z, giving your answer in radians to two decimal places. © Find the complex number w for which zw = 25 + 35i, giving your answer in the form p + ig, where p and q are real.tisfy the © the complex numbers z and ws simultaneous equations 22+ iw = -1,2-w=3+3i. a Use algebra to find z, giving your answer in the form a + ib, where @ and bare real. b Calculate arg z, giving your answer radians to two decimal places. ‘The complex number z satisfies the ation 2 i equation 7 ~ i, AER, a Show that 2 — 2 34001 + a) 1+ b In the case when A = 1, find |zl and arg z, ‘The complex number 2 is given by z=-2+2i. a Find the modulus and argument of z. b Find the modulus and argument of 3, © Show on an Argand diagram the points A, Band C representing the complex } respectively. d State the value of ZACB. numbers z, 4 and z + ‘The complex numbers z, and z, are given by = v3 +iandz=1-i. a Show, on an Argand diagram, points representing the complex numbers 7;, zand 2; + 2. bb Express 1 a + ib, where a and b are real numbers. and 2, each in the form © Find the values of the real numbers Aand B such that A + 8 Beata ®@ ‘The complex numbers z and w are given byz are real numbers. Given that z+ w a find the value of A and the value of B. bb For these values of A and B, find tan[arg(w — 2). a Given that z= 2 Pa=3-4i — i, show that b Hence, or otherwise, find the roots, 2, and Z, of the equation (+i =3-4i © Show points representing z; and z) on a single Argand diagram. dl Deduce that lz; ~ z)1 = 2/5. € Find the value of arg(z, + 22). a Find the roots of the equation 2 + 4z +7 = 0, giving your answers in the form p + i/g, where p and q are integers. b Show these roots on an Argand diagram. ¢ Find for each root, i the modulus, ii the argument, in radians, giving your answers to three significant figures. 3iven that © and that z and w are ‘complex numbers, solve the simultaneous equations z — iw = 2,z— Aw=1-%, giving your answers in the form a + ib, where a, bEB, and a and b are functions of A. eo Given that z, = 5 ~ 2i, a evaluate Iz}1, giving your answer as a surd, b find, in radians to two decimal places, arg 2; Given also that 2; is a root of the equation P ~ 10z + c= 0, where cis a real number, find the value of c.© Using your values of p and q, plot the The complex numbers z and w are given points corresponding to %, Z2 and z; on by z= 5 "hand w = iz. an Argand diagram. a Obtain z and w in the form p + ig, d Verify that 27, + z; — z; is real and find where p and g are real numbers. its value. b Show points representing z and w ona single Argand diagram ED) Given that z = 1 + v3iand that ‘The origin O and the points representing Weo2+2i, find zand w are the vertices of a triangle. ¢ Show that this triangle is isosceles and state the angle between the equal sid @ © the exact value for the modulus of w. a win the form a + ib, where a, bE R, b the argument of w, On an Argand diagram, the point A represents z and the point B represents w. a Find the modulus and argument of each of the complex numbers 7, and 7). Draw the Argand diagram, showing the points A and B. b Plot the points representing z), z, and 71 + zona single Argand diagram, € Find the distance AB, giving your answer as a simplified surd, © Deduce from your diagram that an (32) = 142 tan (3) = 1 +02. © @® Tre solutions of the equation 2+ 62+ 25 = Oare 7 and z,, where 0 < arg z,< wand —7
q. arg 2 = arg z— arg 7". © Verify, for z = V3 — i, that b Given that 7, 7; = 7, find the value of p Display z, z* and % on a single Argand and the value of g. diagram.€ Find a quadratic equation with roots zand Z* in the form ax? + bx + ¢=0, where a, b and care real constants to be found. 147i 443 a Find the modulus and argument of z. z b Write down the modulus and argument of z. In an Argand diagram, the points A and B represent 1 + 7i and 4 + 3i respectively and O is the origin. The quadrilateral OABC is a parallelogram. ¢ Find the complex number represented by the point C. i Calculate the area of the parallelogt | — i, where A is a positive, Given that 2+ 2! real constaitt, a show that 2=(4 +1) + i(3— Given also that tan (arg z) = 4, calculate b the value of A, © the value of Iz. oe ‘The complex numbers z; = 2 + 2i and z, = 1 + 3i are represented on an Argand diagram by the points P and Q respectively. a Display z; and z, on the same Argand diagram. b Find the exact values of 14, Iza! and the length of PQ. Hence show that © AOPQ, where O is the origin, is right- angled. Given that OPQR is a rectangle in the Argand diagram, d find the complex number z, represented by the point R. The complex number z is given by 2= (1 + 3i)(p + qi), where p and q are real and p > 0. 2 3 a show that p + 2g = 0. Given also that Iz| = 10V2, b find the value of p and the value of g. Given that arg z © Write down the valueofargz. @ € The complex numbers z; and z, are given by 2, =5+i,2=2-3i. a Show points representing z, and z; on an Argand diagram. b Find the modulus of 4 ~ z». € Find the complex number 7! in the form a + ib, where a and b are rational numbers. d Hence find the argument of 4 giving your answer in radians to three significant figures. € Determine the values of the real constants p and q such that 2i. oe z= a + ib, where a and bare real and non- zero. a Find 2 and 4 in terms of a and b, giving each answer in the form x + iy, where x and y are real. b Show that I = a2 +B. © Find tan(arg 2) and tan (arg), in terms of and b. On an Argand diagram the point P represents 7? and the point Q represents and 0 the origin. Using your answer to ¢, or otherwise, show that if P, O and Qare collinear, then 3a? = B’.@® Starting with x = 1.5, apply the Newton— Raphson procedure once to f(@) = x3 — 3 to obtain a better approximation to the cube root of 3, giving your answer to three decimal places. @D fle) = 2 + x- 4. The equation f() = 0 has a root a in the interval (1, 2]. Use linear interpolation on the values at the end points of this interval to find an approximation to a. Given that the equation x$ — x — 1 = 0 has a root near 1.3, apply the Newton-Raphson procedure once to f(z) = x*~ x ~ 1 to obtain a better approximation to this root, giving your answer to three decimal places. x 120+ 7. ® t= a Use differentiation to find f’ @). The equation f(x) = 0 has a root @ in the interval 1
0, lies on the parabola with equation y? = 4ar. The tangent and normal at P cut the x-axis at the points T and N respectively. Prove PT that Fe ‘The point P lies on the parabola with equation 5* = 4ax, where a is a positive constant. ©, where cis a positive t e a Show that an equation of the tangent to the parabola P (ap?, 2ap), p > 0, is py =x + ap. ‘The tangents at the points P (ap?, Zap) and Q(ag?, 2aq) (p + 4, p > 0, q > 0) meet at the point N. b Find the coordinates of N. Given further that N lies on the line with equation y = 4a, ¢ find pin terms of g. oe The point P (at®, 2at), t # 0 lies on the parabola with equation y? = 4ax, where a is a positive constant. a Show that an equation of the normal to. the parabola at P is The normal to the parabola at P meets the parabola again at Q. b Find, in terms of f, the coordinates of Q. a Show that the normal to the rectangular hyperbola xy = c%, at the point P(ct, §) t+ 0, has equation y = 2x + : ~ cf. ‘The normal to the hyperbola at P meets the hyperbola again at the point Q. 'b Find, in terms of f, the coordinates of the point Q. Given that the mid-point of PQ is (X, Y) and that # +1, ¢ show that ¥ = The rectangular hyperbola C has equation xy = 2, where cis a positive constant. a Show that the tangent to C at the point (ep &) nas equation py = ~x + 2p. ‘The point Q has coordinates Q(ca, 7 qt P.The tangents to C at P and Q meet at N. Given that p + q #0, b show that the y-coordinate of N is The line joining N to the origin O is perpendicular to the chord PQ. ¢ Find the numerical value of Pg’. @ ‘The point P lies on the rectangular hyperbola xy = c?, where c is a positive constant. a Show that an equation of the tangent to rlep, £ the hyperbola at the point P (@ pr p> 0, isyp® + x = 2cp. This tangent at P cuts the x-axis at the point S. b Write down the coordinates of S. € Find an expression, in terms of p, for the length of PS. The normal at P cuts the x-axis at the point R. Given that the area of ARPS is 4c, d find, in terms of ¢, the coordinates of the point P. The curve C has equation y? = 4ax, where isa positive constant. a Show that an equation of the normal to. Cat the point P (ap, 2ap), (p # 0) is y+ px = 2ap + ap’. ‘The normal at P meets C again at the point Q (aq’, 29). b Find q in terms of p. Given that the mid-point of PQ has . 125, _ 3 coordinates (184, ~ 3a), © use your answer to b, or otherwise, to find the value of p. The parabola C has equation y? = 32x. @ Write down the coordinates of the focus SofC. b Write down the equation of the directrix of C. ‘The points P (2, 8) and Q (82, ~32) lie on C. © Show that the line joining P and Q goes through S. The tangent to C at P and the tangent to C at Qintersect at the point D. Show that D lies on the directrix of C. cayAfter completing this chapter you should be able to © add, subtract and multiply matrices © find inverses of 2 x 2 matrices * represent some geometrical transformations with 2 x 2 matrices * use matrices to solve linear simultaneous equations. Matrix algebra This chapter will give a brief introduction to matrices and some of their applications. Matrix algebra is used in many branches of mathematics, especially those areas where large volumes of data are handled as the tules for combining matrices are easily implemented ‘on computers. Transformation matrices are often used) to create 3-D computer graphics. You will meet some further properties and applications if you study FP3.Matrix algebra [aq You can find the dimension of a matrix. A matrix can be thought of as an array of numbers (a collection of numbers set out in a table) and they come in different shapes and sizes. You can describe these different shapes and sizes in terms of the dimension of the matrix. This is given by two numbers n and min the form n x m (read as n by m), where nis the number of rows (horizontal or across the page) and m is the number of columns (vertical or down the page) in the matrix. @ Ann X mmatrix has n rows and m columns. Hl Matrices are usually denoted in bold print with a capital letter e.g. A, M ete. Give the dimensions of the following matrices al? } bl 0 2), a2 (hah 0-3 yj} There are two rows. 3. \__1____________ There are two columns, The dimension io 2 X 2. @ i) 3) —_—_—-— — There is just one row. aa | eee al —— There are two rows. (A The dimension is therefore 2X1. | | There is just one column. ey teed 4 = ‘There are three rows. o -3! ct There are two columns. The dimension is therefore 3 X 2. |CHAPTER 4 | 4,2 You can add and subtract matrices of the same dimension. ‘To add or subtract matrices you simply add or subtract the corresponding elements of the (2 1 él Given that A + B = x 3 ;, find the values of the constants a, b, x and y. -co(2 3)4 (6 -1\_(8 ae ca (F 3\+ ol (? a You can describe the position of individual element Comparing corresponding elemento: Cena thee fener aa EST ESTE column e.g. second row, ESHER ELE first column of Cis x. By _- considering the equivalent clements in A and B you can form the equation for x. First row, second column: Second row, first columnMatrix algebra ren 1) Describe the dimensions of these matrices. a(4 3) »(3) | aq 2 3) [1 0 0) e@ -1) {0 1 °| 001 2) For the matrices a-(? Ye 9), 13 1) find aA+Cc bB-A cA+B-C 3) For the matrices a-(}),B-a -1), €=(-1 1 0), D=-@1 -0, E=( ‘Fe 1 3), find where possible: a Ate DA-E ec F-D+C a B+C eF-@+0 f£A-F gC-(F-D). 7 al ees) 0 : 4) Given that( ¢ 7)— (1 ¢)=(8 2), find the values ofthe constants a b, cand d. . 1 & ake ‘(ee thi = fi the il fa, b 7 5 Given that . \+(¢ . F (63 ‘, ind the values of a, b and c. - 5 3) [a by (7 1 6! Given that} —1 *[e a}-[e Ofnd he values of, 54a f 2 Ve fl 4)CHAPTER 4 4,3, You can multiply a matrix by a scalar (number). HA scalar is something that isn’t a matrix. In FP1 a scalar will simply be a number. To multiply a matrix by a number you simply multiply each element of the matrix by that number. eee Note that 2A gives the same anower as A + A. b $B=(5 O -2) $x0=0 {a a} ®=(5 3) Given that A + 2B = C, find the values of the constants a, b and c. 6 6 =(r ch arm=ca(* Jeet 4)=(2 © 102i iO 3! Toe Comparing corresponding elements: First row, first column: at2x1=63a-4 This gives four equations. Firot row second columm 0+ 2b=6=> b=3 per or ee ee ae awl the relevant ones. Second row, second column: 2+2X3=c>c=6Matrix algebra aCrren ., _{2 0) 1) 1 Forthe matrices A= (2 3) B= 1)? find a 3A DIA © 2B. : 01 0 2\_(o 7 2 Find the value ofk and the value ofx so mat (2 aed o 4) ( of : . a O\_4f1 c\_{ 3 3) 3 Hind the values of a 6, cand dso that 24 ° aft -(3 . } = . Sc iio Find the values of 6, cand dso that (> a}-as (3 al 3 2k’ Kk 5] Find the value of k so that a ): (24) ~ (i | 44 You can multiply matrices together. I Whilst the rule for adding and subtracting matrices appears to be quite natural, it is the rule for multiplying matrices that gives them their useful properties. I The basic operation consists of multiplying each element in the row of the left hand matrix by each corresponding element in the column of the right hand matrix and adding the results together. The number of columns in the left hand matrix must equal the number of rows in the right hand matrix. The product will then have the same number of rows as the left hand matrix and the same number of columns as the right hand matrix. So if 1nis from the number of rows in A. AxB =C 24 kis from the number of columns in B. Dimensions: (7X m) x (im XK) (n XR) oo ‘These numbers must be the same. Given that A= (37 ‘ana B First calculate the dimensions of AB______——~._ The number of rows is two from here. ~The number of columns is one from here.a= (5 )(3)-)-— 1X (-3) + (-2)X2=-7 3x (-3)+4x2= 0 AB = (2) Example Ed -{ Given tha ven that A = (~} a AB b BA. a Aisa 2X2 matrix and Bis a 2X2 matrix 60 they can be multiplied and the product will be a 2X2 matrix ao =(—! alle i= (2 bl B SO -2) Nc 1) X440xX0= 4 (1) x14+0x(-2)=-1 o=2X443X0=8 d=2X14+3%x (-2)=-4 50 AB = ( sal & -4, b BAwill also be a 2X 2 at bale G3) | sxepe ——— Two numbers are required in the answer. The top number is the total of the first row of A multiplied by the first column of B. The bottom row is the total of the second row of A multiplied by the second column of B. “Matrices with the same number of rows and columns are called square matrices. The product of two square matrices of the same size will always give a square matrix of the same size. This time there are four numbers to be found. iis the total of the first row multiplied by the first column. bis the total of the first row multiplied by the second column. cis the total of the second row multiplied by the first column. dis the total of the second row multiplied by the second colurnn. First row times first column =-2 First row times second column ~ 4x0+1%3=3 Second row times second column — 0x04 (-2)x3 = -6 Second row times first column “Ox (-1) + (-2)x2= =4_ Matrix algebra Notice that in Example 6 you could not find BA because the dimensions do not allow you to use the rule. In the case of square matrices though, like Example 7, you will always be able to find both AB and BA, and usually these answers will be different. (The technical term for this is to say that matrix multiplication is not commutative.) This means that, when you are finding the product of two matrices, it is very important that you place them in the correct order. Given that A= (1-1 2),B=(8 ~2)and@= (3), determine whether or not the following products can be evaluated and, if they can, find the product. a AB b BC © AC aca © BCA a AxB enions: (1 X lice eed Aelia eh AC ‘These two numbers are not the same Not: possible so the product can not be found. b BXxC “These two numbers are the same so Dimensions: (1X 2) (2X 1) nee -0ft)= ex the product is possible and the answer will be 1 x1. NB Matrix B is written (3 —2) ie. without a comma, to avoid confusion Row tir i ie ana TD pet i with the coordinates of a point (3, —2). c Axe Dimensione: (1X 3) X (2X1) cia GAG HIS OSES HEE TIGL INGEST Not possible — * 50 the product can not be found. a cxA Dimensions: (2 X 1) x (1X 3) ESSE Tae the product will have dimension 2 X 3. first row X second column gives 4 x (—1) = first row x third column gives 4 x 2 = 8 \ second row x first column gives 5 x 1 = 5 Soca = (4 = 2) second row x second column gives 5 x (—1) = 5-5 10 second row x third column gives 5 x 2 = 10 ( = (ent 4x(-1) 4x2 first row X first column gives 4X 1 = 4 5x1 Sx(-1) 4 |CHAPTER 4 e BXC XA Dimensions: (1X 2) X (2X1) X (1X3) — These two numbers are the same AND ~ these two numbers are the same. So the product is possible and will This can be calculated as B(CA) or (BC)A. avecanention aise. We shall do both. 4-4 8 5 B(CA) = (3-2) . cay =@ -a(t —8 2) Using CA from part d =(2 -2 4) (BC)A=(2)(1 -1 3)=(2 -2 4) Using BC from part b. When evaluating the product of three or more matrices, provided the order is kept the same it does not matter which product pair is evaluated first. (We say that matrix multiplication is associative.) Sometimes a little forward planning can make the evaluation simpler; (BC)A was easier than B(CA) in this case. Some calculators will evaluate matrix products and, whilst this facility may be useful to you in the FP1 exami you should know how to calculate the product by hand as ‘sometimes you will be given matrices containing unknown letters. wae a-{landp-o 2 Given that BA =(0), find AB in terms of a. BA =(b 2 ")= (b+ 2 BA will be a 1 x Imatrix. a So BA = (0) implies that b = 2a a6 = (Te a=( ? a AB will be a 2 x 2 matrix. a ab 2a Substituting b = 2a gives AB = (ee 2) 2a? 2aMatrix algebra ren 1 Given the dimensions of the following matrices: Matrix A 2x2 Dimension Give the dimensions of these matrix products. a BA b DE © cD aED ets a Find these products. \-3) o(! i 0 3\ ah 2 3 4-1 -2, -1- Ot The matrix A= (~) 3 )ana the matrix B (i i ’}. Find Aemeans AXA. a AB a ‘The matrices A, B and C are given by _ (2 _{31 a a=(2, pa(} 3) e-c3 -2. Determine whether or not the following products are possible and find the products of those that are. a AB b AC © BC ad BA e CA f CB Be 6) 9 Find in terms of a [7 NG to} Bd 3 2\(x -2 Find in terms of (_3 2 3h ‘The matrix A = (; i}. o4 Find ‘You might be asked to prove this ad formula for A‘ in FP1 using induction from Chapter 6. bas © Suggest a form for A‘ : i 0 The matrix A (f } matrix A= (5 5 a Find, in terms of a and b, the matrix A’. Given that A? = 3A, b find the value of a.CHAPTER 4 9) A-C1 3), »-(i} c-(é 2) 0 0-3) Find a BAC bac 1 10, A= =(3 -2 -3). 2 Find a ABA b BAB You may be familiar with the idea of a transformation from GCSE. There you may have met transformations such as rotations, translations, reflections and enlargements. There is more about these specific types of transformations in Section 4.6. HM Atransformation moves all the points (x, y) in a plane according to some rule. You can describe a transformation in terms of its effect on the position vector ( } this is simply the vector from the point (0, 0) to the point (x, y). You call the new point to which (3) is moved the image of &, } ‘The three transformations , T and U are defined as follows. Find the images of the point (2, 3) under each of these transformations. .(2e-9\, v(*) (%), aty yl \—a Substituting x = 2 and y ie. the point (1, 5) 2x ~ y gives 1 and into x + y gives 5. 3 into ie. the point (6, 2) Substitute x =2 and y = 3 into the S formulae, ie. the point (6, —4) REivhen terete) A linear transformation has the special properties that the transformation only involves linear expressions of x and y (so U in Example 10 is not linear) and that the origin (0, 0) is not moved by the transformation (so $ in Example 10 is not linear).Matrix algebra m Ali \sformation T has thi ies. near transformation T has these properties. ioe ke Bs : definitions of linear transformations 1 1{) = er(*), where kis a constant. ‘work and is not typical of questions hy, y: ‘you will find in the FP1 examination. 2al(5)+(F)]-1(R) +) en For the transformations S, T and U from Example 10, use the vectors ( 7 ) and lea to show that $ and U do not satisfy property 2 and prove that T satisfies both properties. a(-1+4)_( 3 These two points are not o-1) \-1 the same so $ is not linear. wio=a[_j)+9(~4) =| “ide (a} lel (2) So S is not a linear transformation. vend} fae? wf eal) Bea) 2 These are not the same so U is not a linear transformation. To prove that T is a linear transformation you need to use general points, not specific vectors like we did for $ and U. ell =2)- Fe 8)- Tere (f)— (755 $) means ky kx + ky. that the new top number is twice. the old top number minus the old _J/a= ¥) bottom number. xty So T satisfies property 1 q (3) i (4) Ee a #2 %] a (48 + x) — [+ ye] ‘Compare LHS and wu! MWe Nt Ye m+ He + [+ ye) RHS of property 2 using the rule for T. 123) +1(%4= aa si)e 2x, ye) = (En + Dit y. w Ye uty Xe + Yo m+ x2 + Tht ye So T satisfies property 2. Therefore T is a linear transformation.CHAPTER 4 Any linear transformation can be represented by a matrix. ax + by cx + dy The linear transformation (3) ( svce(2 G)(3)= (S28). ) can be represented by the matrix M. ( @ 8) cd) Find matrices to represent these linear transformations, 2), zy | (*)+(52 ) al?) (2 ows arty a Transformation T io equivalent: to 1 (?} , (te + 24) y) — \3x + Oy 60 the matrix is te oh 0 b Traneformation V io equivalent to [x\_, (ox — 2y v:{) ( xy yh Next y | 60 the matrix is (9 2) Example {E} ‘Write the transformation in the form [ax + by) lex + dy! Use the coefficients of x and y to form the matrix. \Write the transformation in the form [ax + by) lex + dy Use the coefficients of x and y to form the matrix. ‘The square $ has coordinates (1, 1), (3, 1), (3, 3) and (1, 3). Find the vertices of the image of $ under the transformation given by the matrix M = ( The vertices of S can be represented by the matrix 125 1 ) ( ij 5 3) To find the image of S you need the matrix product: (3 il 1 2) 2 ‘Write each point as a column vector so (1, 3) becomes (} Then combine all four column vectors into a single 2 x 4 matrix. Use the usual rule for multiplying matrices._ Matrix algebra 1| Which of the following are not linear transformations? eG) eal)-e) eebhBer) xg) enGhe) ee) 2) Identify which of these are linear transformations and give their matrix representations. Give reasons to explain why the other transformations are not linear. as(z}-(%-7) wn (3)-(22)) eu (3)~[3] av-(3)-(%) ew:(3)-(2] 3| Identify which of these are linear transformations and give their matrix representations. Give reasons to explain why the other transformations are not linear. 2 3 as-(*}~() »r-(*)— (>) yl Ny y x - 0 evft)(e9) — ave}-(¢ 3] es yO, ow(3)-() yl Ny 4) Find matrix representations for these linear transformations. “Ghs*) eG) a) 5! The triangle T has vertices at (~1, 1), (2, 3) and (5, 1). Find the vertices of the image of T under the transformations represented by these matrices. 4) 0 -2' o 4) “\2 0CHAPTER 4 6 The square $ has vertices at (~1, 0), (0, 1), (1, 0) and (0,1). Find the vertices of the image of § under the transformations represented by these matrices. “(6 5) of) elt 4) 4.6 You can use matrices to represent rotations, reflections and enlargements. I In GCSE you may have met some simple transformations such as rotations, reflections, enlargements and translations. @ =A translation is not a linear transformation (since the origin moves) but all the others are and in this section we shall see how to represent them using matrices. To identify the matrix representing a particular transformation you should consider the effect of the matrix or the transformation on two simple vectors (2) (sometimes denoted 0! asi) and ( ) (sometimes denoted by j). ” Given any matrix M (2 ») you can see that 1 a 0 b m(e)= (2) am m(t)= (2) so the first column of M gives the image of (’) and the second column of M gives the : 0 of . image ct °) You can use this information to identify the transformation represented by a matrix. In FP'I you should be able to identify matrices representing the following linear transformations. Rotation about (0, 0) of angles that are multiples of 45°. Enlargement centre (0, 0) of scale factor kk + 0, KE fi). Reflection _in coordinate axes or the lines y = +x. Identity the matrix ( O| scaled I and does not carry out any transformation. (This is equivalent to multiplying by 1 in arithmetic.)Matrix algebra Describe fully the geometrical transformations represented by these matrices. sos} eC 4) (44) a y A diagram is very useful. Use it to (°) show the images of (}) and (°) 1 M() : ae! Under the action or( I this becomes Oo 3, The images of () (blue) and (°) (red) are in the same directions as the original vectors but 3 times as long. ( 0 ) This indicates an enlargement. When describing an enlargement you 3 should state the scale factor and the ( 3 ) centre (always (0, 0)).. ay The transformation is therefore an enlargement, scale factor 3 and centre (O, 0) Draw a diagram showing the images si os. 1 4 The vector (1) has moved to (—}) This could be due to a reflection in x= ora rotation of 180°. (2) The vector (9) has moved to (_9} o “This could be due to a reflection in Oo z y= 0 or a rotation of 180°. Since the (9) Same transformation has caused both ‘movements it must be a rotation. ‘This traneformation is therefore a rotation of er 160" (anticlockwise) about (0, 0). be described as an enlargement centre (0, ©) scale factor —1.CHAPTER 4 When describing a rotation you should ¥ state the angle, direction (anticlockwise is positive) and centre (always (0, 0)). ‘The diagram shows that (}) has moved ( to (_9} This could be due to a rotation 0 of —90°or a reflection in y | (0 ® ( ) Also (0) has moved to (1). This could be due to a rotation of +90°or a reflection in y = —x. Since the same transformation has carried out both This transformation ie therefore a reflection ‘movements it must be a reflection. in the line y = —x. ‘To describe a reflection you must state the mirror line. Parts b and ¢ of this example are often confused so a carefully drawn diagram is useful. Another way of identifying a reflection is to look im = (2 2) the determinant at the determinant of the matrix (see Section 4.8). (e 5) an of M is given by ad ~ be. If this is negative the transformation involves a reflection. Sere 15] ‘ind matrices to represent these transformations. a Rotation of 45° anticlockwise about (0, 0). b Reflection in the y-axis. © Enlargement centre (0, 0) of scale factor ~2. Draw a diagram showing the images of the original vectors after the rotation of 45°.Consider the image of (d} | Similarly the image of (°) is 1| \v2 = val yak Vv! Therefore the matrix representing this transformation The matrix representing a reflection =] ol o1 be -2x(!)=(~2) 0 oO o image of (°) -2x (°\= ( | 1 1} \-2, ‘The matrix representing an enlargement: of scale factor —2 about (0, 0)is { im - in the line x = O is ( Image of Matrix algebra Remember that cos 45° = 1 and sin 45° ‘ aka Remember that the image of () gives the first column and the image of ( second. il The diagram shows that (t) moves to (3) but () remains unchanged. The first column is ie a the image of () and the second column is (} the image 0) a(t} To enlarge a vector by scale factor k you simply multiply the vector by k. Image of Q) as the first column andCHAPTER 4 eens 1 Describe fully the geometrical transformations represented by these matrices. A i | b ic s) = ( oO. 0-1 1 0 1 0) 2) Describe fully the geometrical transformations represented by these matrices. a (° 4) (! ° b ce 3 1 0 \o 1 3) Describe fully the geometrical transformations represented by these matrices. ER 40 tet a v2 VE ) : (: ) len i (04 ala i} v2 v2 4) Find the matrix that represents these transformations. a Rotation of 90° clockwise about (0, 0). b Reflection in the x-axis. € Enlargement centre (0, 0) scale factor 2. 5 Find the matrix that represents these transformations. a Enlargement scale factor ~4 centre (0, 0). b Reflection in the line y = x. © Rotation about (0, 0) of 135° anticlockwise. 4,7 You can use matrix products to represent combinations of transformations. i The points A(1, 0), B(O, 1) and C(2, 0) are the vertices of a triangle T. The triangle Tis rotated through 90° anticlockwise about (0, 0) and then the image 7” is reflected in the line y = x to obtain the triangle 7”. a On separate diagrams sketch T, 1’ and T". b Find the matrix P such that P(T) = T’ and the matrix Q such that Q(T") = T". © By forming a suitable matrix product find the matrix R such that R(T’) = T". d_ Describe the single geometrical transformation represented by R. € Find the single matrix representing a reflection in the line y = x followed by a rotation of 90° anticlockwise about (0, 0). f Describe the geometrical effect of the matrix found in part e as a single transformation.y 2 b Note that OA moves to OA’ and then to OA" and OB moves to OB" and then ‘to OB". P represents a rotation of 90° about (0,0) and so ed 1 0 Q represents a reflection in y = x so (ey 1 & 6 P(N) =T' and Q(T’) = So Q(P(N) = 1" And therefore QP = R Matrix algebra This is the original triangle 7. Rotate 90° about (0, 0) so that (2) moves to{7} ane (f} moves to (7) This gives 7. Reflect T' in y = x s0 that (°) moves to()=@(9) This gives T*. ~GhG)-) ~ Gana) (0) Substitute P(7) for T'. Q(P(D) can be written as QP(T)CHAPTER 4 Use the usual rules of matrix multiplication = ( 1 2 met in Section 4.4. Draw a diagram showing the images of (8) (bie) ana (9) (red) under R. () line) and (°) moves to (_©] which confirms unmoved (since it lies on the mirror R represents a refection in the x-axis ‘or the line y =O the reflection. The transformations are the other way around, The required matrix is PQ. ra =(° ae 5} To transform with Q first followed by P you 1 oli o. calculate PQ. 1 ft yt Draw @ diagram showing the images 1 0 of () (blue) and 2 ) (red). oO, ¥ PQ represents a reflection in the 0) The fact that (2) does not move suggests that the y-axis is @ mirror line. Consideration 1 ' of (3) confirms this. y-avis (or the line x = 0). In general: the matrix PQ represents the traneformation represented by Q followed by the transformation represented by P. You can combine more than two transformations in a similar fashion.Matrix algebra ‘The matrices P = (3 a e-(5 ana Fi 3) represent three transformations. Find the matrix representing the transformation represented by R, followed by Q followed by P and give a geometrical interpretation of this combined transformation. The combined transformation is given by 1 1\f1 2\f5 7 FaR = )( ( ) R first so start on the right with R, the iG allo ‘| 1-2, nears ay richer a 133 7 You can choose whether to multiply PQ ( )I | of QR first but the order PQR must be 2 7-1 —2) maintained. =(¢ 1)" "= 7 A This is PQ To interpret this transformation, draw a diagram y Note that 1) Vang (0). (1 (3) (-S)ana(f) = (9) which points to a rotation. 0 ¥ PGR represents a rotation of 90" (ie. 90° clockwise) about (0, 0). ert va-(2 Se 0-1 20 0-1 ( 1 e-( 2) Find these matrix products and describe the single transformation represented by the product. a AB b BA © AC an ecRA 2 A = rotation of 90° anticlockwise about (0, 0) B = rotation of 180° about (0, 0) C = reflection in the x-axis D = reflection in the y-axis a Find matrix representations of each of the four transformations A, B, C and D. b Use matrix products to identify the single geometric transformation represented by each of these combinations. Reflection in the x-axis followed by a rotation of 180°about (0, 0). Rotation of 180° about (0, 0) followed by a reflection in the x-axis. Reflection in the y-axis followed by reflection in the x-axis. Reflection in the y-axis followed by rotation of 90° about (0, 0). v__ Rotation of 180° about (0, 0) followed by a second rotation of 180° about (0, 0). vi_ Reflection in the x-axis followed by rotation of 90° about (0, 0) followed by a reflection in the y-axis. vii Reflection in the y-axis followed by rotation of 180° about (0, 0) followed by a reflection in the x-axis. Use a matrix product to find the single geometric transformation represented by a rotation of 270) anticlockwise about (0, 0) followed by a refection in the a-axis. Use matrices to show that a refection in the y-axis followed by a reflection in the line y = —x is equivalent to a rotation of 90° anticlockwise about (0, 0). 1 1 ‘The matrix Ris given by} V2 v2 1 2 vB a Find R*. b Describe the geometric transformation represented by R2. © Hence describe the geometric transformation represented by R. d Write down R*. 5 2) 1-2) P-( 3 -}e-( 3 s) The transformation represented by the matrix R is the result of the transformation. represented by the matrix P followed by the transformation represented by the matrix Q. a FindR. b Give a geometrical interpretation of the transformation represented by R. Ss -7 s a ing the three transformations in the order B, followed by A, followed by Ca single transformation is obtained. Find a matrix representation of this transformation and interpret it geometrically. 1 S| (; -1p@ ‘Matrices P, Q and R represent three transformations. By combining the three transformations in the order R, followed by Q, followed by P a single transformation is obtained. Find a matrix representation of this transformation and interpret it geometrically. Ra 0 }®Matis algtra 48 You can find the inverse of a 2 x 2 matrix where it exists. m A= (? 5) then A The value of ad ~ be is called cod the determinant of A and written det(A). and then AA“! = A“"A . a-(2 8), det(a) = ad ~ be so Ata 1 ( ~~ det(A) : a) -< a Notice that if det(A) = 0 you will not be able to find A" because stray is not defined, in such cases we say A is singular. If det(A) = 0, then A is a singular matrix and A ' cannot be found. If det(A) # 0, then A is a non-singular matrix and A ‘exists. See _{ 3 2 21 13 a-| 1 i} ( ihe ( ) For each matrix A, B and C, determine whether or not the matrix is singular. If the matrix is non-singular, find its inverse. =| 3 2) « det(a) =3 x 1-2 (1) Use the determinant formula with alee a=3,b=2,c=-1andd=1. det(A) = 5 ‘Swap a and d and change the signs of band c. A ‘can be left in either form, 2 2) = (B) =2X1-1X2= a (f 1/20 eH) : Remember if det(B) =0 then B is singular. So B lo singular and B can not be found Note that a determinant can be < 0 ¢ (e 2). devo) = 10-3 x2 2 0, This is non-zero and 60 C is a non-singular vatrik, matrix ‘Swap a and d and change the signs of b and c. Then multiply by 1 daCHAPTER 4 Some calculators ind inverse matrices and you may wish to use your calculator to check your answer. You may be asked to find the inverse of a matrix in algebraic form. _{ 4 p+2' a4 | When p =x the matrix A is singular. a Find the value of x. Given that p +x b find A+ leaving your answer in terms of p. a det(A) = 4(5 — p) — (p + 2)(-1) det(A) = 12 - 4p + p+ 2=14-3p Ai singular so det(A) = O 4-3x=0 - =4 ams A= 1_((3-p bets aor en mp —(p+ 2) 4 Write A= Swap @ and d and change the signs of b and c. P and Q are non-singular matrices. Prove that (PQ) "= Q 'P en Let C = (PQ) PPQC = Pl then (PQ) ee a $0 QC @ac=ar! So IC= QP 50 C=Q"'P If Aand Bare non singular matrices then (AB) Use the definition of inverse AXA=1=AA4, Multiply on the left by P. ember P-'P = I, 1Q = Qand Multiply on the left by Q7. Using Q:'Q BAY,A and Bare 2 x 2 non-singular matrices such that BAB = I. a Prove that A= BB! 2 4 ik b find the matrix A such that BAB = 1. Given that B = ( a BAB = | B'BAB= Bl ie. (B°B)AB = BO co AB=B" ABB = B' Bo Al= B46 And hence A= BB | la 8) 60 det(B) = 2x 3-5X1=1 Fe (| Matrix algebra ‘Multiply on the left by B-'. ‘Multiply on the right by B-? and remember BB" = I. First find B!. Use the result from part a and matrix multiplication to find A. 1| Determine which of these matrices are singular and which are non-singular. For those that are non-singular find the inverse matrix. 3-1 aoa “(5 3) a | i 2 5 3 als 3) . tf 2) 2) Find the value of a for which these matrices are singular. ee aoa a+2 1-a a lta 2 fae eo) c aRA a 10 uu Find inverses of these matrices. a ita’ +a 24a) Bil 2a es) -a -b, i) find B. a Given that AB = G, find an expression for B. 3 6 7 — = Given further that A = (7 j)anac ( 7 ) naw a Given that BAC = B, where B is a non-singular matrix, find an expression for A. & 3) b When C find A. en ( 2am : 2-1 _[4 7-8 The matrix A= | 7 Jana ap=( 3 3) Find the matrix B. aa i -1)\ ‘The matrix B {and AB=|—8 9}. Find the matrix A. \-2 -1 The matrix A = | where a and b are non-zero constants. a FindA'. ‘The matrix B ( al and the matrix X is given by B = XA. b Find X. ‘The matrix A = IF 2a) and the matrix B b 2b a Find det(A) and det(B). b Find AB. ‘The non-singular matrices A and B are commutative (i.e. AB = BA) and ABA = B. a Prove that A? = 1 a A cd Given that A = a a b show that a = dand b= c. ) , by considering a matrix B of the form_Mouis letra _ | 49 You can use inverse matrices to reverse the effect of a linear transformation. Example 2) 4-1 ‘The triangle T has vertices at A, B and C. The matrix M ( fe with vertices at (4, 3), (4, 10) and (~4, ~3). a Find the coordinates of the points A, B and C. b Sketch triangles Tand 7’. © Show that the area of 7” transforms T to the triangle 7” area of T multiplied by det(M). a. Let the coordinates of Tbe given by the Write the coordinates of T'as a matrix X. 2x3 matiix(%1 9 3) Vi Ye the welt 4 = Then X is a 2 ¥ 3 matrix too. \s 10 -3, MX = M { Multiply on the left by M-? and remember that M"'M 60 x=M { M és Use the usual rules to find M", if 1 4 4 -4 So x= H-3 ae 10 >) le. X= 1? iS ) Itis usually simpler to keep the 0 8 oO fraction (5 in this case) outside and multiply through with it inthe final 2 - step. a = gs = Write down the coordinates of A, B Sons (\. 0) Ba (24) and es (=1 0) and C from the columns of X. b y 24) A sketch need not be accurate but ie it should show the relative positions of the points and have some points marked to give a sense of scale. Tor F Triangle TCHAPTER 4 Triangle T” AveaofT=1x2xK4=4 2 = Use bh with b = 2 and h = 4. Area of = 2X BX7= 28 det(M) =1X4—-3xX(-1)=7 Use 4bh with b = 8 and h = 7 (the distance from (4, 3) to (4, 10)). So area of T' = area of TX det(M) since 26 = 4X7 rat ia 1 : (0 -1 The matrix R \ i 3) a Give a geometrical interpretation of the transformation represented by R. b Find R. © Give a geometrical interpretation of the transformation represented by R~. -1 0 ( 0-1 i. Give a geometrical interpretation of the transformation represented by 8. ii Show that S* = 1 iii Give a geometrical interpretation of the transformation represented by S-. a The matrix b The matrix T = ( i Give a geometrical interpretation of the transformation represented by T. ii Show that T? iii. Give a geometrical interpretation of the transformation represented by T € Calculate det(S) and det(T) and comment on their values in the light of the transformations they represent.Matrix algebra 3) The matrix A represents a reflection of 270° about (0, 0). a Find the matrix n the line y = x and the matrix B represents a rotation =BA and interpret it geometrically. b Find C+ and give a geometrical interpretation of the transformation represented by € ¢ Find the matrix D = AB and interpret it geometrically. d Find D- and give a geometrical interpretation of the transformation represented by D-. 4,10 You can use the determinant of a matrix to determine the a the transformation. 1 Asyou will have seen in Example 22, for a transformation represented by a matrix M. Area of image = Area of object x |det(M)| Sometimes the determinant is negative. In this formula you just use the value of the Example FE} determinant. That is what Idet(M) means. The triangle OAB where A is (2, 0) and B is (1, 3) is transformed by the matrix M- where p is a positive constant. a fi \d the coordinates of the images of A and B in terms of p. b Given that the area of the image of OAB is 30, find the value of p. © Sketch OAB and its image for the case p = ~3 and comment. Since (0, 0) is always a AB unchanged by a matrix P il 21 transformation, you only need 3 2No 3} to consider A and B, Write their coordinates as column vectors 2p PO) in a2 X 2 matrix 69 | So the image of A fo (2p, 6) and the image eee Ore Re ee of Bis (p -6,9) image of B. b B t Sketch triangle OAB and h i use tbh, ° zo 1 2 det(M) = 2p — (-2)X B= 2pt+6 So the area of OAB xX2xX3=3 Find det(M). area of object X |det(M)| Use the determinant as an area scale factor. Area of imagCHAPTER 4 If we had not been told that 20 = 3(2p + 6) p> 0 then we could have 10 = —(2p + 6) leading to 10=2p+6 p= 8. Remember if 4=2p det(M) <0 this tells you that en the transformation represented So p=2 by M involves a reflection. ¢ When p = —3 then, using the answer to part a, the image of Ais (6, 6) the image of B is (—9, 9) B kh I 3 ° so 9,9) A singular matrix will always transform points onto a straight (-6, 6) Ie. OE The image of OAB is a straight line with equation y = —x This is confirmed by considering the determinant det(M) = (-3) X 2-3 Xx (-2)=-6+ 6=0 ened alUler angaias the transformation has ‘squashed’ the triangle matrices. into a otreight line of zero area ren 1. The matrix A la es) is used to transform the rectangle R with vertices at the points (0, 0), (0, 1), (4, 1) and (4, 0). a Find the coordinates of the vertices of the image of R. b Calculate the area of the image of R.2 5 Matrix algebra The triangle T has vertices at the points (~3.5, 2.5), (—16, 10) and (~7, 4). a Find the coordinates of the vertices of T under the transformation given by the matrix _f-1-1 a ( 3 3} b Show that the area of the image of Tis 7.5. c Hence find the area of T. ‘The rectangle R has vertices at the points (—1, 0), (0, ~3), (4, 0) and (3, 3). P34) where ais constant, a Find, in terms of a, the coordinates of the vertices of the image of R under the transformation given by A. b Find det(A), leaving your answer in terms of a. Given that the area of the image of R is 75 € find the positive value of a. eG the-(a ahe-G i A rectangle of ares rectangle when X it ‘The matrix A = ( Sem’ is transformed by the matrix X .Find the area of the image of the aP baQ cR a RQ © QR £ RP ‘The triangle T has area 6 cm? and is transformed by the matrix (3 constant, into triangle T’. a Find det(A) in terms of a. Given that the area of T’ is 36 cm? b find the possible values of a. 5 mple EZ) Use an inverse matrix to solve the simultaneous equations 2x ay Sx + 6 Write these equations as a matrix 2 ~3}(*)-( 5) product. The LHS is a matrix M, made = Up of the coefficients of x and y from the equations, and a vector,CHAPTER 4 Find the inverse of M. Multiply on the left by Mt. Since MM = . ) a | It is usually easier to leave a fraction ° such as —} outside until the last step. (*) a ( =2) of the calculation. yl \-3, 1. Use inverse matrices to solve the following simultaneous equations a 7x+3y=6 b 4x-y =-1 —Sx — 2) 2) Use inverse matrices to solve the following simultaneous equations a4x-y =11 b Sx + 2y 3x4 2y=0 3x4 4y NT Roce 1 1) The matrix A = ( ) transforms the triangle PQR into the triangle with coordinates 6, ~2), (4,4), 8). Find the coordinates of F, Qand R. 2) The matrix A 5 Find the matrix B.aia( she-(s ahe-G i} 7 Matrix algebra ‘The matrices A, B and C represent three transformations, By combining the three transformations in the order A, followed by B, followed by C, a simple single transformation is obtained which is represented by the matrix R. a find R, b Give a geometrical interpretation of the transformation represented by R. ¢ Write down the matrix R’. ‘The matrix ¥ represents a rotation of 90° about (0, 0). a Find ¥. ‘The matrices A and B are such that AB = Y. Given that B = (@ 2) b find A. © Simplify ABABABAB. ‘The matrix R represents a reflection in the x-axis and the matrix E represents an. enlargement of scale factor 2 centre (0, 0). a Find the matrix C = ER and interpret it geometrically. b Find € * and give a geometrical interpretation of the transformation represented by € 7 ‘The quadrilateral R of area 4 cm? is transformed to R’ by the matrix P = , where pisaconstant. a Find det(P) in terms of p. Given that the area of R’= 12 cm? (pas P' b find the possible values of p. The matrix A= (5 a where a and b are non-zero constants. a Find A. ‘The matrix Y (a 2) and the matrix X is given by XA = ¥. b Find x. ‘The 2 x 2, non-singular matrices A, B and X satisfy XB = BA. a Find an expression for X. 5 b Given that A = fa ala ind B= 2 1) sna x.RA Summary of key points 1 2 Ann X m matrix has n rows and m columns. The transforma ion represented by the matrix product ABC means first do the transformation represented by C second do the transformation represented by B third do the transformation represented by A. ‘The matrix 1 ( a A is the identity matrix or transformation. It does not change a matrix or an object. b The determinant of amatrix A= ("7 )is det(A) = ad — be. 1 det(A) The inverse of a matrix A ( Nisa 1 dAfter completing this chapter you will know how to: use the result for the sum of the first n natural * use the results for the sum of the squares, and the sum of the cubes, of the first n natural numbers, Yyrand yo 3 respectively . use the results for S71, So, So? and = fet eat to = sum series where the general term is a polynomial in r of degree at most 3, e.g. }> (28 + 2 — 3r-+ 6), 2 You will also be more familiar with the > notation and know = yal Vi By ~ ww the result Ati Series, and particularly infinite series, form an important part of the study of mathematics. Functions that you may be familiar with, such as sin x, Inx, e* can all be written as infinite series: for example, ae =) et = of functions can be expressed algebraically, as infinite series, is both a unifying aspect of the subject and a very useful one. . The fact that a wide range This chapter primarily considers a special group of finite series, the sums of powers of the first n natural numbers, n BS the notation ’3-" to mean Sor fork = 0, 1, 2and 3. BBM the sum of a series. iI The sigma notation is a very useful, and concise way, to define a series. It makes further study of series more manageable. I Asyou saw in Book C1, Chapter 6, where the ” notation was first introduced, YoU = Uy + Uy + Up + Uy +. Uy where U, is a function of r. IH Itis also used to mean ‘the sum of the series’. For example, > PaPrPr Pete but you will also see that > (= B(n+ Qn +1), the sum of the series (this result will be proved in Chapter 6). Series such as arithmetic series, geometric series and the binomial series, which you have 24 already studied, can all be written in > notation: for example, ar! ‘sums up’ the geometric series a+ ar + ar? + ..bart a Write out the series defined by the following a )-er-1) a ‘These expressions represent a general 2r+ term in the series. a c De +2) a a or) =@xi-n+@x2-HN+@xs—-N+@x4—-N tut (Qn NST StS 474.4 (2n-1) ‘The same series can be eS expressed in different b r+!) =@xorntexitn+ ex2tt ways in)" notation, =O F(2XS H+ + [An + =14+ 345474 ...4 (20-1) © Lew +2) =(? +2) + (2? +2) + (H+ 2) 4 (424.2) 4+ .4(r? +2) 4 =S+GFN+ Ot... + (1? +2)Series Write these series using the > notation. a3+S+7+9+11413 b2+5+10+..+ 0+ 1) © 1X2HZXSH3X 4TH (N= 2ln— 1) a 3454749 +i +13 is the oum of the odd numbers from 3 to 13. ‘An odd number may be represented by (2r — 1), where rie an integer. The values of r corresponding to 3 and 13 are 2 and 7 respectively, 90 the series can be written as) > (2r — 1), ‘ Equally >_(2r + 1) could be used. c ail b 245+10+..4(? +1)= > (241) ca e The general term of the series IX 24+ 2XB4+3X 4+... + (n—2)(n— 1) can be written as r(r + 1). Using thio expression the firet term corresponds to r = 1 and the final term corresponds to = (n= 2),60.me can write So (r+ 1): rite out each of the following as a sum of terms, and hence calculate the sum of the series. 5 0 air by ae m= a 1 5 4 a Sp? +3) e Doors 1 £ S216 - 4% fat Fa a 2! Write cach of the following as a sum of terms, showing the first three terms and the last term. a Yor -1) b Yer +1) : s © 0-40+4) a) pe +3) fat imcHaPick 5 [3 In each part of this question write out, as a sum of terms, the two series defined by Sf); for 0 1 example, in part ¢, write out the series )“r? and )“r. Hence, state whether the given, ma fF statements relating their sums are true or not. a ert 1)=)Gr-2) b aa ae © de [S) foi am = et ela ms Y © Nertrg ae 4 m1 m1 = 4 ay r= |4_ Express these series using > notation. a3+4+5+6+7+8+9410 b 1484 27 4 64+ 125 + 216 + 343 + 512 © 11421435 +... + Qn? +3) 114+ 21435 +... + Qn? 4n + 5) © 3XS4+5X7+ 7X94... + @r- Y@rt I) +... tok terms. The sum of the first n natural numbers, 1 + 2 +3 + ... + n, is an arithmetic series of n terms, with a= Vand 1=n. Using the formula 5 = 5(a + 1) for the sum of an arithmetic series, ) r= 2(n + 1). 2 1275 — 20 x 21 = 1275 - 210 = 1065.met Show that )| k= 2N?-N-10,N=3 ms =)+]_4x5 e 2 = (2N - 1)(N — 10) = 2N- N-10 rt 1 Use the result for ) “r to calculate % fat ° ayy by*y = Be oe ye ibe ah Given that y a show that 2 + 1 — 1056 = 0 b find the value of n. 528, meat a Find Sk m1 mt b Hence show that )) = Fan yjn>2. 0a ik yk Deo sn — 1) Ys a 8 b Hence evaluate oe ” a Show that )>r— m1 Series In general: Df Yio -\ fo. A.common mistake is to use a0 yn Spe. ‘Substitute n= 2N-1,n=4in5(n +1).cHaPri S All series of the form Yer + b), where a and b are constants, are arithmetic series and so mt their sums can always be found by using the appropriate formula, However, by splitting up ) “(ar + b) into a) “r+ BS “1, you can find such sums using the stmt results for ) “rand “1. amt Show that }“(3r + 2) can be written as 3) “r+ 2) Yert2 =@x14+2)+@x2+3+Gx3+2) = +..4 (3X n+2) =(GXItSNH2+5X5+...+5% g) As 1° = 1 for all positive integers r, the sum of 1 +@+ 2+ 2+..+5 ‘ones, =SXIF245+...+n) (1+141+.. +1) FAltt1+14.. +1) ‘can written as Bum oF noes (1° + 204304 + 2) : Soe =de-d where DOI= (1141+. + 1) Sn 0 Bum oF n ones Evaluate }(3r + 1) Yer +1)= 2+ ry = 325% 26 3 z + 25 = 975 + 25 = 1000Series a Show that )°(7r~ 4) = 3(n ~ 0). a 0 b Hence calculate §(7r ~ 4). fae 7%{(n +1) — 4n -—_ Itisacommon mistake 2 to write this as 4, not 4n, ES 0, 4 n b rr a= dr- 4- dr -4) = 27160) — 1} - 1247(19) - 1}+-—— Using the result from a. 8725 — 1254 = 74" (In this exercise use the results for “rand 1.) 1) Calculate the sum of the series: a Ser y b Se -7) c Ye + 21) mt mi a 2] Show that . an an a dire = an +7) b> — 4) = n(10n - 3) ne © Art 3=(94 2046 — AD (4p + 8)= An + Yn 2) a 4 pa f 3) a Show that } “(4r— 5) = 2k? ~ 3k. ma b Find the smallest value of k for which )_( 4r~ 5) > 4850.[4 Given that u, = ar + band 1u, = 5(7n + 1), find the constants a and b. et [5) a Show that )7(1 + 3A) = 24n? - 2n-1,n>1 a » b Hence calculate ) “(1 + 31). had [6) Show that ) "(4 — 51) = -@k + (Sk + 1), k=0 mt ‘The standard results Veer ee ee este Hat N@n+ YD ma Seas eae n= Bae De = will be used without proof in this Chapter. Proofs will be given in Chapter 6. 0 2s Evaluate a So 1? by =m mo Aone, ee | = Bao + 1)(80 +1) — Bue + 1)(38 + 1) = 22.140 — 2470 = 19670 Oe —FeeSeries | Replacing n by 2n in - gine 12n + 1). gine 1)(2n + 1) = = Bean + (an + j- a a When you have Ben + M240 +1) — (n+ HP been asked to find . a general result for =n + (+1) sum it is good practice to test it for small values of ‘n. It will not prove that you are correct, (2n + 1)(7n +1) =1(3)(8) = 4¥ _ dutif one value of 6 n does not work, you know that your result is incorrect. n 6 2(6)16) = 257 5 gen +1)(7n + 1) = eCrran 1) Verify that )> ma = Gl + 2n + 1) is true form = 1, 2 and 3. 2| a By writing out each series, evaluate _F for m= 1, 2, 3 and 4. mi b By writing out each series, evaluate for n= 1, 2, 3 and 4, © What do you notice about the corresponding results for each value of 1 ? wo wo x0 3) Using the appropriate formula, evaluate a Sr? ob Yo oe Soe = oh & 4) Use the formula for “7? or }7P to find the sum of mao a P+ 2+ Peet. + $2? b 23+ 334454... + 408 © 262 + 27 + 28? + 297 +... + 100% A124 24H K+ IP e +2? +384 ..+ (n- 1)?5 For each of the following series write down, in terms of n, the sum, giving the result in its simplest form mast ay aye bye yr a S ayr eS nro. i raat 6 Show that a)? =hn— N@n® + Sn +6) £@ + D4n + 1) 20 Fa thee thas Sy MG * BEE * fn (8, a Show that Sr} = n°(2n + 17. b By writing out the series for "(on', show wnat) 20" = a Se et © Show that 1° + 3° + 5° +. + 2 — 1)* can be written as » Yen. 1 Hence show that the sum of the cubes of the first odd natural numbers, 14 384 S84. + Qn DS is Qn? ~ 1). In Example 5 we saw that Yort 2) can be written as yr + Dae This demonstrates the mam at use of two general rules relating to sigma notation. addition rte + = Suey = Tree multiple rule ya Dy MI These rules mean that more complicated sums tke) “(or + br+ and rt Soar + be? + r+ a) can be found using the results for "1, J or and ye. m1 fet fet ret mt where kis a constantSeries : 4 a Show that Le +r-2)= Fin + ayn 1) b Deduce the sum of the series 4 + 10 + 18 + 28 + 40 +... + 418 24+r-2)=) 2+ =) F Le : TE5 pag y Using the addition rule and the multiple rule, any number of . * terms can be ‘split up’. =Yetry ratte - ‘ - an ° = Gln + NAH +1) + H(n +1) — 2H Use the results ee aa = Bln + (2n + 1) + B(n + 1) - 12] Glatt Sn + 1+ Bn + B— 12] gar + 6n- 8] = Bl? + 3n— 4] = Bn tayn-1) b et r-2)= 044410414 284404. +418 S04+10 +18 +26 +40 +... +418 ea Substitute n = 20 in 2(n + 4(n — 1). = Beeo + 4\(20 - 1) = 3040 2g a Find the sum of the series }r(r + 3)(2r 0 b Hence calculate }r(r + 3)(2r~ 1)cHaPia S a Sines 3)(2r— 1) = rer? +51 = ae + sor oat = 2 2 HF + B2(n + Nn +1) - 3B (n +1) Using the results for op and) -3r) First multiply out the brackets. Use the addition and multiple rules. = OED ann + 1) + 82n +1) - 9] y = ute Vian? + on - 4] b ree B)(2r 1 rn 10 Ps Soret By2r— 1) —P Orie + B21) ‘= ont = 40X41 x 5316 _ 10 X 11x 426 6 = 1453040 — 7810 = 1445230 Substitute n = 40 and n = 10 in the result fora. irate 1 Use the formulae for ye ye, Serena yon, where appropriate, to find A 0 a Sone -1) b Sve +4) c Ye +3) a Sw —2). ant a m= mh 2) Use the formulae for )“r', ) “r2, and ) “y, where appropriate, to find moet Pa 20 away by er» «Paso, at ri a giving your answer in its simplest form. 3) a Write out } (7+ 1) asa sum, showing at least the first three terms and the final term. a b Use the results for “rand ) “r? to calculate at At 1X242X343K44+4XS4+5X64...4 60% 61.Series 4) a Show that }(r + 2)(r-+ 8) = Yor + 12n + 41). m1 0 b Hence calculate “(r+ 2)(r-+ 8). fad 7 7 “ _ (n= Ian + 1Y(n- + 2) 5) a Show that De pes y- eer b Hence find the sum of the series 13 X14 15 + 14K 15 X 16 + 15 X 16 X 174... + 44K 45 x 46, 6/ Find the following sums, and check your results for the cases # = 1, 2 and 3. ayy ber 1% e Sore ay i crt a 7) @ Show that) Ar 1) = For — NGn + 2). b Deduce the sum of 1 x 2? + 2.x 3%+3 x 4% ... + 30x 31%, Bia Show that) “¢ ~ fr + 1) = 22n + 5(n~ 1). b Hence sum the series 1x 3+ 2X 4+3X5+... + 35x 37. 2 9) @ Write out the series defined by ye + 31), and hence find its sum. 20 ad v that S> =" 14n? 1b Show that Ds we + 3A) = 5 (an? + 181 + 3). ¢ By substituting the appropriate value of n into the formula in b, check that your answer for a is correct. 10 Find the sum of the series 1 X 1+ 2X3+3%S +... ton terms, Nite Rontge 1) @ Write down the frst three terms and the last term of the series given by )@r + 3). b Find the sum of this series. ae Verify that your result in b is correct for the cases n = 1, 2 and 3. so 0 2\ Finda So@r+5) b&b S21) a 3) Given that }°U, m+ An, b Deduce an expression for U,. ¢ Find )°U,. 20 4! Evaluate Yer -) mtNv 10 | 12 13 14 15! an | Show that)” Find) “r°¢r — 3), m= 20 Show that )“(2r— 1)? 2m 1 6,2 3 (lon a Show that) + 2)= Ent Qn + 7). b Using this result , or otherwise, find in terms of n, the sum of Blog2 + dlog2? + Slog2* +... + (n+ 2)log2" a Show that) (2 —r~ 1) = Bin ~ 2yn + 2). a 0 b Hence calculate) (r? — r~ 1). a Show that) er += Fas yor+ns d. 8 b Hence calculate) > 1(2r? + 1). % Find a Y1@r-1) b r+ 28r+5) Yer ar +1). a ai ao a Show that) tr +1) =40 + In + 2). “ b Hence calculate )“1(r + 1). a a Show that rr + 1)(r-+ 2) = Fin + Il + 2Vn + 3). mt b Hence evaluate 3X 4X5 +45 X6+5X6X7 +... + 40% 41 x 42, a Show that }“r120 -n+ a b Hence sum the series (2 — 1) + 2(2n — 3) + 3(2n— 5) +... bn gut Dn + V, a Show that when mis even, B-2B+R-..- P= B+ Br B+ ute 16 (15 + 284 38+ Bb Hence show that, for n even, I’ — 28 + 38 — © Deduce the sum of 18 = 28 + 3 =... ~ 408. ae + 1)(an + b), where a and b are constants to be found, = -™@n +3)Series Summary of key points 1 The notation) “U, defines the series U, + U; + + Uy, where U, is a general term in the series; it also is used to mean ‘sum the series’. 2. If the series is summed from r = k tor n, then 3. The sums of powers, k, of the first natural numbers, for the cases k = 0, 1, 2 and 3 are © Sesser. tien ” ‘These should be learnt. Dora14+24+3+..+n=2n+1) Yrs 243+. takin 1)(2n + 1) E(B B+2+3?+...4+ m= ons 1F 4 Ifthe general term is a more complicated function of r, the sum can be ‘split up’ so that some, or all, of the above basic results can be used. For example: a Die tar —2r+5) Sona -ayir+s b der Dera) Yer srt 2-23 8+ 81+ DaAfter completing this chapter you will know how to: * use the method of mathematical induction to prove general statements which involve positive integers. Proof by mathematical induction The structure of a proof by induction can be compared witha line of falling dominoes. If the first domino falls, then so will its neighbour, until all the dominoes fall. The first example of rigour in proof by induction was a proof that the sum of the first n odd numbers is n?. This was given by Francesco Maurolico (1494-1575).Proof by mathematical induction 6.1 You can obtain a proof for the summation of a series, using induction. | n=1; 1=1 n=2; 1+3=4 n=3; 14+3+5=9 n=4 14+3+5+7=16 n= 5; 14+34+5+7+9=25 n=6, 1434547494 11=36 ete. Looking at the summations above we can easily spot a general result which is true. General Statement: If you add up the first n positive odd numbers you obtain the squared number 1?, i Ve This general statement can be written as: 143454+7494...4+ (20-1) - Using the series notation learned in Chapter § the general statement can be also rewritten as: The diagram below is also a pictorial representation of the result for n = 1, 2, 3,4 and n= 5. eoce . eeee . eooe . econ ° eoee =4 This may give more insight about why the general statement is true. In this chapter, however, you will lear a more formal method of proof, known as proof by induction, to prove general statements, such as the one above, which apply to positive integer values, n, where > 1. Proof by mathematical induction usually consists of the following four steps: Step 1: Basis: Prove the general statement is true form = 1. Step 2: Assumption: Assume the general statement is true for n = k. Step 3: Inductive: Show that the general statement is then true for n = k + 1. Step 4: Conclusion: The general statement is then true for all positive integers, 1.Prove by the method of mathematical induction, that, for nZ*, )(@r ~ 1) =n? aI 1 n=tLHS = S@r—1) =2() -1 =A RHS = =1 As LHS = RHS, the summation formula is true for n= 1. Assume that the summation formula ig true for n = k, f ie. Ye-n= 2, a ~ cad With n= k-+ 1 terms the summation formula a becomes: atl Sen atteret. fGen, Simattt mee rj FEED DT ngisnetes tantam = ke + (2k+ 1-1) = k* + (2k+ 2-1) = (k+ 1% Therefore, summation formula is true when n=kt1. If the summation formula is true for n= k then it is shown to be true for n= k-+ 1. As the result is true for n = 1, it is now also true for all n= 1 and n€Z* by mathematical jones | =a © In step two, it has been assumed (but not yet been proved) that the general statement is true i forn =k ie.) @r~ 1) =k, a * The assumption has then been used in step three fo show that the general statement is true when n = k + 1, At this point we are now able to apply step four, the conclusion.Proof by mathematical induction * In step one, it has been found that the general statement is true for n = 1. * In step two and step three, it has been found that for all n€ Z*, Cer — 1) is assumed true for n = k-> )°@r— 1) is true forn =k +10) a Et * By applying step one and (*), it can be deduced that the general statement is true for n = 2. By applying (*) again it can be deduced that the general statement is true for 1 = 3. By continuing to apply (*) it can be deduced that the general statement is true form = 3,n=4,=5,..., n=kn=k-+1,n=k+ 2, etc, Therefore the general statement is true for all n > 1 and nEZ* by mathematical induction. « Therefore the general statement }”(2r~ 1) = n has been proved to be true for any positive a Si Vee integer by mathematical induction. * Note that proof by induction is not used, however, to derive a general statement from first principles. Proof by induction is used to check whether a given statement is true. “ Prove by the method of mathematical induction, that, forn€Z*, )r? = In(n + Qn + Y. 7 1. Basis step: Substitute n= 1 into both the LHS and RHS Of the formula to check to see if the formula works for n= 1. As LHS = RHS, the summation formula ie true for n= 1. Asoume that the oummation formula is true for n= k t _ 2, Assumption step: ie. Dor in this step you assume that the result given = in the question is true for n ~ k. SMk + 1)(2k + 1).With n = k + 1 terms the summation formula becomes: |S inductive step: Thee OY = EKk + (2k +1) + (kK +1? = 2k + NM 2k+ 1) + 6k +1) =4 2 = 2k + 2k + k + 6k + 6] = Dk + 2k? + 7k + 6] = E(k + 1)(k + 2)(2k + 3) = LK + NK+1E AKT +1) Therefore, summation formula te true when n = k-+ 1. Hf the summation formula to true for n = k, then it ie shown to be true for n = k + 1. As the reoult is true for n= 1, it is now also true for all n> 1 and n€Z* by mathematical induction. Prove by the method of mathematical induction, that, for nZ*, 12" = 2[1 + (a ~ N24. 1 BA BE Sell = » n=t LHS = )or2r=12)'=2 = RHS = 2[1 + (1 - 12°] = 2(1) = 2 As LHS = RHS, the summation formula is true for Assume that the summation formula io true for n = ft te. Soran = 2p + (k- 24, = With n= k + | terms the summation formula becomes: (2!) + 2(22) + B02") +... + M2) + (K+ AEA rl = t+ (k= 124] + (kt Nak! 22420 12+e!_ __ = 2+ (k— 12k OF (K+ art! +(k-1 4k + 12k! + Bhar {+ kaR+ 9) = Qi + ((k+ 1) - Nek*] Therefore, summation formula is true when n= K+ 1. If the summation formula is true for n = k, then it ie shown to be true for n= k-+ 1. As the result is true for n = 1, it is now also true for all n > 1 and n€ Z* by mathematical induction.Proof by mathematical induction ren Prove by the method of mathematical induction, the following statements for n€ Z*. 1 Sor dn + 2 i mi 3 Snr 1) = dain + yn) 4 (1X6) + 2X7) 4+ BX B)4 + nln $$) =Inln + 1K + 8) s Srar- 1) =n + 1) 6 Yer 1% Ian? ~ 1) zi a 7 Yee meine a 9 Srey a4 y-1 a Prove, by induction that 32” + 11 is divisible by 4 for all positive integers n. Let f(n) = 3% + tt, where nEZ*. fl) = 3% + 11 = 9 + tl = 20, which s #(i) is divisible by 4 when n Assume that for n = k, fk) 3* + tis divisible by 4 for kEZ* 7 fk) = 4 Pt 0 = (3) +11 = 42.8) f(k + 1)= #(k) + 4(2(379) Therefore f(n) is divisible by 4 when n= k +1. IF f(r) is divisible by 4 when n = k, then it: has been shown that f(n) is also divisibie by 4 when n = k + 1. As f(r) Is divisible by 4 when n = 1, f(n) 6 also divicible by 4 for all n> 1 and n€Z* by mathematical induction. | WAKE |S. i VesProve, by induction that #3 ~ 7n + 9 is divisible by 3 for all integers for n€Z'. Let f(r) f(l) 1. Basis step f(r) is divisible by 3 when n Agoume that for n 2. Assumption step f(k) = K — 7k + 9s divisible by 3 for kKEZ*. 3. Inductive step 7 fk 4+ 1) = (k+ 12 -7k+ +9 P+ Se + e+ 1 - 7+ +S = 2 HE 7 49. Use of Binomial Theorem or =P +32 + 3kK+1- 2-749 1 prising ar res B+ 3K ~ 4k +3 = 5 + fk -+ 1) - (8) = [+ BE = 4k + 3) — [ - 7k + 9] BE + 3k-6 2) As both f(K) and 3(02 + k ~ 2) | are divisible by 3 then the sum ok +1) =f + BE + k- 2) of these two must also be 7 divisible by 3. Therefore f(r) io divisible by 3 when n= k+1. 4, Conclusion step IF f(n) ie divisible by 3 when _shown that f(r) is algo divisible by 3 when n= k +1. As f(r) is divisible by 3 when nt = 1, f(n) is algo divisible by 3 for all n = dana neZzt by mathematical induction. k, then it has been Example 6 illustrates a more demanding problem involving divisibility.7. A(t) = 1 + 12 = 133, divisible by 133. “A Basisstep f(r) fo divieible by 135 when n = 1 f(k) = 1+! + 12s divisible by 135 for ke Z* fk +1) = ttt tg zee aah Teta eee (kt) + 1440122 1) \ (k + 1) — #(k) = [100 +9) + 144012 = 1) — kt + 12-1) joc) + 148012" ") joc +t) + 10(122*-') + 13312 1 12277) + 133 (12% Fk + 1) = FR) + Op + 1221] + 13301221) = f(k) + 10F(k) + 133012") Prove, by induction that 11"'1 + 12®"~ ‘is divisible by 133 for all positive integers 1 } Let f(n) = tt +12" 5 where ne Z* a n€Z* by mathematical induction.‘ In Questions 1-8, use the method of mathematical induction to prove the following statements for ne Z*. 1 8"— Lisdivisible by 7 2 3% — 1 is divisible by 8 3. S" +9" + 2is divisible by 4 4 2 — 1 is divisible by 15 5 3% 1 4 1 isdivisible by 4 6 n+ Gr? + Bris divisible by 3 7 n° + Snis divisible by 6 8 2".3?" — 1 is divisible by 17 9 fi) = 13" 6 nEZ'. a Express for KE Z", {(k + 1) — 6f(K) in terms of k, simplifying your answer. 'b Use the method of mathematical induction to prove that f(2) is divisible by 7 for all nez! 10 g(v) = 5% — 6n+8,nEz*. a Express for kE 2”, g(k + 1) ~ 25g(k) in terms of k, simplifying your answer. b Use the method of mathematical induction to prove that g(n) is divisible by 9 for all nEz*. 11. Use the method of mathematical induction to prove that 8" — 3” is divisible by 5 for all n€ Z*. 12 Use the method of mathematical induction to prove that 3” ' ? + 8n — 9 is divisible by 8 for allnez*. 13. Use the method of mathemat allnez*. induction to prove that 2 + 3°? is divisi le by 5 for 6.3. You can use mathemati recurrence relation. | induction to produce a proof for a general term of a In Cote 1, you saw recurrence formulae which allowed you to generate successive terms of a sequence. eee A sequence can be described by the recurrence formula +4n>Lu=1 Une = a Find the first five terms of the sequence. b Show that that the general statement , = 3" ~ 2, 11> 1, gives the same first five terms of the sequence.Proof by mathematical induction Substituting n Substituting n Substituting n = Sun +4 Buy +4 Bu, +4 3) +4= 25 3(25) +4 ae SuesHENt i iestti pasties E The first five terms of the sequence are 1, 7, 25, 79, 241 b General statement, u, = 3" — 2,21 Substituting n Substituting n — Substituting n= 3: 6 Substituting n = 4; Substituting n The first five terms of the sequence are algo 1, 7, 25, 79, 241. Example 8 demonstrates how to apply the method of proof by induction to show that the general statement 1, = 3" — 2 is true for the recurrence formula u,, = 31, + 4, with first term u, = 1and n> 1. Given that u,,, = 3u, +4, m4 = 1, prove by induction that 1, = 3" — 2. n= tu = 3'—2=1as given. n=2u, BZ ard up = Bu + 4 = 3(\) + 4=7, from the recurrence relation. So u, is true when n = 1 and is also true when n = 2. Accume that for n= kuy = 3°— 2ie tue for kEZ”. Cee Then 41 = Buy, +4 oo . [= 3-2) +4 ogee = GS, sry 1 : Therefore, the general statement, u, is true when n= k +1. c L If u, is true when n = k, then it has been shown that u, is also true when n = k +1. As u, is true for n=1 and n= 2 then 4. Conclusion step u, io true for all n> 1 and n€Z* by mathematical induction, In the basis step of the proof, the general statement was checked for both n= 1 and n = 2. This , from the general statement. because the first application of the recurrence formula 1,,;— 3u,, + 4 yields u by using the first given term u, = 1._ Given that i, . = Sty. 1 ~ 6t4y t= 13, 4, = 35, prove by Induction that uy, = 2"'1 43", [maby 2+oe=1,asgven a ats step tu 26, ao given The first application of a 2 BP OF from the neners| ckatement__ hte Fecurrence formula ~~ 97, from the general statement, __ yields uy by using b 5(35) —6(13) uy; and u,. So check (BRE ty S lle ~ Oy = I) OW) a ctatement for — & [es vee Setanitheeomrencerelakior E 4 So u,, is true when n = 1, n = 2 and also when n = 3. + = = —— — kand n= k +1, that both 2. Assumption step Aeoume that, for n T= 24 Band Us = DEFIH 4 getitt Ue K+ 2 + Bk+? are true for ke Zt Bee 4 Bt ra BH 29) + 5A — 62) — 6B} = 522) + BETA) — B22) 2G Tae + er) r aie aa ee 26" ght etig geteti Cae i 34 4 iz Then ue +2 = Su+1— Guy 3. Inductive step |___ This the same as CL ga 2th 4 3°+1 with aS nneplaced by k + 1. Therefore, the general statement, uy, is true when 4. Conclusion step | k +1 then it has been shown 1,n= 2 and n= 3, then u, 6 If u, is true when n = kand ‘that u, 9 also true when 2. As ue for true for all n> 1and n€Z* by mathematical induction. In Example 9, each term of the sequence depends upon the two previous terms. Therefore u, was assumed to be true for both # = kand n= k +1. L_ Given that , , ; = Su + 4, 4, = 4, prove by induction that 4, = 5” ~ 1. 2. Given that u,. 1 = 2u, + 5, tn = 3, prove by induction that 1, = 2”*? ~ 5. 14, 3. Given that 4, , = Su, ~ 8, ty = 3, prove by induction that 1, =Proof by mathematical induction 4 Given that 1, , , = 3u, + 1, u, = L, prove by induction that 1, = aeh 5 Given that 2 = Sty 1 ~ 6ty, t = 1, ty = S prove by induction that u, = 3" ~ 2", 6 Given that 1, . 2 = 614, — 9ty, th = 1, uy = 0, prove by induction that 1, = (7 — 2)3"— 1, 7 Given that 2 = 7ty 1 ~ 10d, 4 = 1, th = 8, prove by induction that 1, = 25" 1)—2"- |, 8 Given that 1, . 2 = 614,41 — 9U,, th = 3, Uz = 36, prove by induction that u,, = (3n ~ 2)3", 64 You can use proof by induction to prove general statements involving matrix multiplication. i Ve Use mathematical induction to prove that (i + = (3 a >) forneZz’, As LHS = RHS, the matrix equation is true for n= 1. Asoume that the matrix equation is true for n = k. 2 Assumption step (2 With n = k-+ 1 the matrix equation becomes If the matrix equation is true for n = k, then it io chown to be true for n= k + 1. Ag the matrix equation is true for n = 1, it is now also true for all n> 1 and n€Z* by mathernatical induction. + (6 =o alo 2) $ o 2) “lo allo 2 bs ay “3:48 7 7 0 a Oo 2 =(549 14+ 2-2(24) O+0 0+225 ee. | Oo mt - Therefore the matrix equation ie true when n= k + 1 | 4.Conclusion stepExample [J] Use mathematical induction to prove tat (7 2 ae Cs a oe "" |) fornez". nev 0-2 9)-(2 9 a = 2 ed RHS = (eae 9(1) -(2 Al —z —() a) +1 -1 4 ‘Ae LHS = RHS, the matrix equation ie true for n ‘Asoume that the matrix equation Is true for n = k. iz. (= ae ey 1 9k } —-1 4 = Sk+1 —— With n = k + 1 the matrix equation becomes ti 14 14 ~ oes : ("ot Et Bek) 2k-Bk-1 0 -Ok + 12k +4 eras 5) eee | K-10 Sk+4. eas Ok + 1) ) —(k+1) Bk+ +4 Therefore the matrix equation is true when n = k + 1. If the matrix equation is true for n = k, then it A.conelusion step is shown to be true for n = k + 1. A the matrix equation is true for n is now also true for all n> land n@Z* by mathematical induction. Ea] Prove by the method of mathematical induction the following statements for n Z*. 1 aya} 2n 3 6 af =f -4n ol ool 1-1 n —2n+1 3 e of. ( 2 9) 4 (5 alee al) 11 2-1 1 2 -3, Qn 1~—4n, 6G T-E)Proof by mathematical induction 1 Prove by induction that 9” — 1 is divisible by 8 forneZ*. 2. The matrix Bis given by B= a Find B? and B’. 'b Hence write down a general statement for B’, for 2°. © Prove, by induction that your answer to part b is correct. 3. Prove by induction that for n€Z°, that )“(Br + 4) = jn@n-+ 11). a 4 Asequence th, th, th ty .. is defined by u, .4 = St, ~3(2"), u = 7. a Find the first four terms of the sequence. b Prove, by induction for n€Z', that 4, = 5" + 2". 5 The matrix A is given by A ; = (821 16n i a Prove by induction that A’ -( a 1g) for mez z ‘The matrix B is given by B = (A")! b Hence find B in terms of n, 6 The function f is defined by f(r) = S**"-' + 1, where neZ*. a Show that f(m + 1) — f(n) = p(5"~}), where is an integer to be determined, b Hence prove by induction that f(7) is divisible by 6. 7 Use the method of mathematical induction to prove that 7" + 4" + 1 is divisible by 6 for all nez*, = 3th B_ Asequence tty, ty, ts, ty. is defined by Wy, a Find the first five terms of the sequence. b Prove, by induction for nZ*, that u,, = 4(, 9 Asequence thy, tz, ty Uy «.. is defined by u,, = 32 + 72") a Show that u,,, ~ 9u,, = A"), where A is an integer to be determined. b Hence prove by induction that u, is divisible by 8 for all positive integers n. 10 Prove by induction, for all positive integers n, that (1X58) + 2X 6) + BX) +... + (a+ 4) = dnl + 12m + 13).Review Exercise Determine whether or not the following products exist. Where the product exists, evaluate the product. Where the product does not exist, give a reason for this. © BAC d CBA. a AB bBA 10 00 (0 1 ando=(j o} Find the values of the constants a and b such that M? + aM + b= 0. Show that A? — 10A + 211 = 0. ab’ O4-(7 jh Find an expression for A, in terms of a, b, ¢ and dso that A? ~ (a+ d)A = AL, where I is the 2 x 2 unit matrix. A= ( _)), where p is a real constant. Given that A is singular, a find the value of p. Given instead that det (A) = 4, b find the value of p. Using the value of p found in b, © show that A? — A = KI, stating the value of the constant k. o(7 |) a FindAt, 251 ~109\ i oS Given that A‘ (33 12)’ b find A‘. A triangle T, of area 18 cm?, is transformed into a triangle 7’ by the matrix A where, _{k k-1 A= fe a wer. a Find det (A), in terms of k. Given that the area of 7” is 198 cm”, b find the possible values of k© A linear transformation from R? > Ris Ba- ‘é : |B ( ‘ a ) defined by p = Nq, where Nis a2 x2 ae : matrix and p, q are 2 X 1 column vectors. Find Given th: 3) i 8) id ae ven that p = [> |when q = [_], an E (; = (0 b AB- BA. that p= (5 ) when a= ( 2 |, finan. Given that C = AB - BA, : : ¢ find, give a geometrical interpretation of the transformation represented by C?. a Find A”, ‘The matrix A represents reflection in the b Find (AB) ', in terms of p. x-axis. ‘The matrix B represents a rotation of 135°, in the anti-clockwise direction, about (0, 0). siven that C= AB, a find the matrix C, Given also that AB = (3 2 ) € find the value of p. @Ma @ -1 b show that C= 1 7-3, a Show that A’ ‘The linear transformation T: R? > is represented by the matrix M, where b Deduce that A?= A '. mee o © Use matrices to solve the simultaneous Ne ay equations ‘The transformation 7 maps the point ary =3, with coordinates (1, 0) to the point Tx — 3y=2. with coordinates (3, 2) and the point with coordinates (2, 1) to the point with coordinates(6, 3). a(S 42 oO 55 al a Find the values of a, b, cand d. a Find A+, b Show that M? =I. b Show that A“BA = (i i ), stating The transformation T maps the point with coordinates (p, q) to the point with the values of the constants A; and Ay, coordinates (8, —3) Find the value of p and the value of q. ( o 1, where p and q are is ‘The linear transformation Tis defined by non-zero constants. (*).(r a Find A |, in terms of p and g. \ a f , 2p 34 The linear transformation Tis represented a Pd } by the matrix C. b find X, in terms of p and q. a Find.The quadrilateral OABC is mapped by T to the quadrilateral OA’B’C’, where the coordinates of 4’, B’ and C’are (0, 3), (10, 15) and (10, 12) respectively. b Find the coordinates of A, B and C. © Sketch the quadrilateral OABC and verify that OABC is a rectangle. 08 -0.4 ~(02 orl and a Find. b Give a geometrical interpretation of the transformation represented by C. ‘The square OXYZ, where the coordinates of Xand ¥ are (0, 3) and (3, 3), is transformed into the quadrilateral OX’Y'Z’, by the transformation represented by C. ¢ Find the coordinates of 2’. 3 11 J )eam(0 2} find the matrices C and D such that Given that A = ( SQ a AC= b DA=B. A linear transformation from B® — Ris defined by the matrix B. Prove that the line with equation .y = mx is mapped onto another line through the origin O under this transformation. d Find the gradient of this second line in terms of m. Referred to an origin O and coordinate axes Ox and Oy, transformations from IR? R? are represented by the matrices L, Mand N, where oO -1 2 0) : ( o}M ( 2) ane w(t a Explain the geometrical effect of the transformations L and M. ‘b Show that LM = N’, ‘The transformation represented by the matrix N consists of a rotation of angle # about O, followed by an enlargement, centre O, with positive scale factor k. ¢ Find the value of @ and the value of k. d Find NS. oe @ A, Band Care 2 x 2 matrices. a Given that AB = AG, and that A is not singular, prove that B = C. b Given that AB = AC, where A = i 4) and B= lo :). find a matrix € whose elements are all non-zero. a Use standard formulae to show that ye 1) = nln? - 1). €® Use standard formulae to show that YMe-y an - Zan + Syn— 1). Use standard formulae to show that Yer - 1p = fatan’— 1. Z) Use standard formulae to show that Youre —3) = Int + yer 29004 3 a ® a Use standard formulae to show that Drer- y= 24 Ayan » mt x b Hence, evaluate )1(2r~ 1).
You might also like
WORKSHEET - Function Transformation Question
PDF
No ratings yet
WORKSHEET - Function Transformation Question
20 pages
Exams International Mathematical Olympiad
PDF
No ratings yet
Exams International Mathematical Olympiad
1 page
4EB1 01 Rms 20180822
PDF
50% (6)
4EB1 01 Rms 20180822
19 pages
4EB1 01R Que 20180620
PDF
18% (11)
4EB1 01R Que 20180620
36 pages
Math Grade 11 FBISE NBF 2024
PDF
100% (2)
Math Grade 11 FBISE NBF 2024
177 pages
Fields of Concentration 2021 2022
PDF
No ratings yet
Fields of Concentration 2021 2022
354 pages
Maths 11 Low Resolution
PDF
No ratings yet
Maths 11 Low Resolution
83 pages
Chemical Coordination
PDF
No ratings yet
Chemical Coordination
30 pages
Edexcel - M3 PDF
PDF
No ratings yet
Edexcel - M3 PDF
186 pages
Further Mathematics Entrance Exam
PDF
No ratings yet
Further Mathematics Entrance Exam
10 pages
Physics For BMAT: Understand The Scope and Level of Challenge of BMAT Physics Questions Practice Questions
PDF
No ratings yet
Physics For BMAT: Understand The Scope and Level of Challenge of BMAT Physics Questions Practice Questions
47 pages
Markscheme Probability
PDF
No ratings yet
Markscheme Probability
12 pages
4EB1 01R Rms 20180822
PDF
29% (7)
4EB1 01R Rms 20180822
19 pages
Edexcel AS and A Level Modular Mathematics Further Pure Mathematics 1 by Keith Pledger
PDF
No ratings yet
Edexcel AS and A Level Modular Mathematics Further Pure Mathematics 1 by Keith Pledger
154 pages
2021 Grammys Predictions - Who Should Win, Who Will Win - The Chronicle
PDF
No ratings yet
2021 Grammys Predictions - Who Should Win, Who Will Win - The Chronicle
8 pages
The Best Albums of 2020
PDF
No ratings yet
The Best Albums of 2020
20 pages
Maths Skills Check
PDF
No ratings yet
Maths Skills Check
7 pages
Examiners' Report: Principal Examiner Feedback
PDF
No ratings yet
Examiners' Report: Principal Examiner Feedback
6 pages
Int GCSE Biology Cell Structure and Stem Cells
PDF
No ratings yet
Int GCSE Biology Cell Structure and Stem Cells
12 pages
Invariant Points and Lines
PDF
No ratings yet
Invariant Points and Lines
14 pages
Section A What? How To Answer? How To Revise?
PDF
No ratings yet
Section A What? How To Answer? How To Revise?
6 pages
Dua Lipa Future Nostalgia' Album Review
PDF
No ratings yet
Dua Lipa Future Nostalgia' Album Review
6 pages
01 As Pure Mathematics Practice Paper A
PDF
No ratings yet
01 As Pure Mathematics Practice Paper A
6 pages
Andrew's Exercise Solutions - Coursera Coursera Introduction To Physiology - Week 1 - Homeostasis and Endocrine System Exam
PDF
No ratings yet
Andrew's Exercise Solutions - Coursera Coursera Introduction To Physiology - Week 1 - Homeostasis and Endocrine System Exam
3 pages
Sample: Core Pure Mathematics
PDF
No ratings yet
Sample: Core Pure Mathematics
11 pages
Alevel Math
PDF
No ratings yet
Alevel Math
10 pages
Ijaz O&A-Level Notes
PDF
No ratings yet
Ijaz O&A-Level Notes
3 pages
Yale College Writing Center WWW - Yale.edu/writing: ODY Aragraph Nalysis
PDF
No ratings yet
Yale College Writing Center WWW - Yale.edu/writing: ODY Aragraph Nalysis
2 pages
Edexcel - S4 PDF
PDF
No ratings yet
Edexcel - S4 PDF
134 pages
The All-Time Most Streamed Artists On Spotify Yardbarker
PDF
No ratings yet
The All-Time Most Streamed Artists On Spotify Yardbarker
1 page
Best Albums in History That Weren't Nominated For AOTY at The Grammys 2
PDF
No ratings yet
Best Albums in History That Weren't Nominated For AOTY at The Grammys 2
1 page
Alpha Workbook 1 of 3
PDF
No ratings yet
Alpha Workbook 1 of 3
58 pages
VCE English Text Response Mini-Guide
PDF
No ratings yet
VCE English Text Response Mini-Guide
8 pages
Further Pure 1 Chapter 5::: The - Formulae
PDF
No ratings yet
Further Pure 1 Chapter 5::: The - Formulae
15 pages
Spotify Charts 3
PDF
No ratings yet
Spotify Charts 3
1 page
V Best Chapter 1.1 Quadratics - ONLINE PDF
PDF
No ratings yet
V Best Chapter 1.1 Quadratics - ONLINE PDF
16 pages
New Edexcel Pure Year 1pdf Compress
PDF
No ratings yet
New Edexcel Pure Year 1pdf Compress
408 pages
9FM0 A Level Maths Papers 1 and 2 Topic Test 2
PDF
No ratings yet
9FM0 A Level Maths Papers 1 and 2 Topic Test 2
31 pages
P1-Revew Exercises
PDF
No ratings yet
P1-Revew Exercises
20 pages
0780 Phy Cgceb 2023 P2
PDF
No ratings yet
0780 Phy Cgceb 2023 P2
6 pages
0580 Probability Teaching Pack v1
PDF
No ratings yet
0580 Probability Teaching Pack v1
41 pages
Logarithms Practice: Created by T. Madas
PDF
100% (1)
Logarithms Practice: Created by T. Madas
22 pages
A Level Further Mathematics For AQA Mechanics Student Book 1 (AS/A Level) Scheme of Work
PDF
100% (1)
A Level Further Mathematics For AQA Mechanics Student Book 1 (AS/A Level) Scheme of Work
23 pages
KS5 "Full Coverage": Integration (Year 2) : (OCR C3 June 2011 Q1i)
PDF
No ratings yet
KS5 "Full Coverage": Integration (Year 2) : (OCR C3 June 2011 Q1i)
32 pages
Tree Diagrams Worksheet, Probability Revision From GCSE Maths Tutor
PDF
100% (1)
Tree Diagrams Worksheet, Probability Revision From GCSE Maths Tutor
5 pages
Cumulative Frequency Worksheet, Information Revision From GCSE Maths Tutor
PDF
No ratings yet
Cumulative Frequency Worksheet, Information Revision From GCSE Maths Tutor
2 pages
QS 015/2 Matriculation Programme Examination Semester I Session 2016/2017
PDF
100% (1)
QS 015/2 Matriculation Programme Examination Semester I Session 2016/2017
28 pages
Untitled
PDF
No ratings yet
Untitled
210 pages
P3 Merged Removed
PDF
No ratings yet
P3 Merged Removed
216 pages
Monday 14 January 2019: Further Pure Mathematics F1
PDF
No ratings yet
Monday 14 January 2019: Further Pure Mathematics F1
32 pages
My Revision Notes_ Edexcel a Level Maths (Pure) -- Sophie Goldie -- Hodder Education Group, London, 2018 -- Hodder Education -- 9781510417410 -- 5848b0a4ab381c46e8dae466db6fce07 -- Anna’s Archive
PDF
100% (2)
My Revision Notes_ Edexcel a Level Maths (Pure) -- Sophie Goldie -- Hodder Education Group, London, 2018 -- Hodder Education -- 9781510417410 -- 5848b0a4ab381c46e8dae466db6fce07 -- Anna’s Archive
236 pages
New A-Level Maths For Edexcel: Year 1 & 2 Exam Practice Workbook
PDF
No ratings yet
New A-Level Maths For Edexcel: Year 1 & 2 Exam Practice Workbook
10 pages
Susan Hooker, Michael Jennings, Jean Littlewood, Bronwen Moran, Laurence Pateman - Edexcel As and A Level Modular Mathematics - Mechanics 3-Heinemann (2009)
PDF
No ratings yet
Susan Hooker, Michael Jennings, Jean Littlewood, Bronwen Moran, Laurence Pateman - Edexcel As and A Level Modular Mathematics - Mechanics 3-Heinemann (2009)
189 pages
T772 GCSE Maths Guide
PDF
No ratings yet
T772 GCSE Maths Guide
13 pages
2) A Level Old Syllabus Vs New Syllabus
PDF
No ratings yet
2) A Level Old Syllabus Vs New Syllabus
5 pages
Histograms Worksheet, Information Handling Revision From GCSE Maths Tutor
PDF
No ratings yet
Histograms Worksheet, Information Handling Revision From GCSE Maths Tutor
3 pages
Circle Coordinate Geometry Exam Questions PDF
PDF
No ratings yet
Circle Coordinate Geometry Exam Questions PDF
106 pages
Edexcel FP1
PDF
No ratings yet
Edexcel FP1
152 pages
Edexcel Pure Mathematics C2 (6664) Specimen Paper Mark Scheme
PDF
No ratings yet
Edexcel Pure Mathematics C2 (6664) Specimen Paper Mark Scheme
4 pages
PM Mathematics Tips
PDF
No ratings yet
PM Mathematics Tips
5 pages
1 Exam Paper - IB - Sequences and Series - Binomial Expansion
PDF
No ratings yet
1 Exam Paper - IB - Sequences and Series - Binomial Expansion
17 pages
Answers: 1.1 Exercise 1A 1.4 Exercise 1D
PDF
No ratings yet
Answers: 1.1 Exercise 1A 1.4 Exercise 1D
30 pages
C4 Integration Worksheet
PDF
No ratings yet
C4 Integration Worksheet
25 pages
Cambridge International AS & A Level: Mathematics 9709/13 May/June 2022
PDF
No ratings yet
Cambridge International AS & A Level: Mathematics 9709/13 May/June 2022
14 pages
Edexcel - M5 PDF
PDF
No ratings yet
Edexcel - M5 PDF
133 pages
O' Level Additional Mathematics Paper 1 (Pure) Jan1971
PDF
100% (2)
O' Level Additional Mathematics Paper 1 (Pure) Jan1971
2 pages
Edexcel IAL Pure Mathematics P2 June 2022 Wma12-01-Que-20220520
PDF
No ratings yet
Edexcel IAL Pure Mathematics P2 June 2022 Wma12-01-Que-20220520
32 pages
Caie As Level Further Maths 9231 Further Pure 1 v2
PDF
No ratings yet
Caie As Level Further Maths 9231 Further Pure 1 v2
16 pages
P3 Topical Pastpapers
PDF
No ratings yet
P3 Topical Pastpapers
73 pages
Madasmaths Hard Question Paper
PDF
100% (1)
Madasmaths Hard Question Paper
6 pages
Paper 1 Mock Exam
PDF
100% (1)
Paper 1 Mock Exam
7 pages
Znotes CAIE A2 Level Further Maths Further Pure 2
PDF
No ratings yet
Znotes CAIE A2 Level Further Maths Further Pure 2
12 pages
Essential Maths - Lauren Gurney David Rayner Paul Williams - 2018 - Elmwood Press - 1906622701 - Anna's Archive
PDF
0% (1)
Essential Maths - Lauren Gurney David Rayner Paul Williams - 2018 - Elmwood Press - 1906622701 - Anna's Archive
336 pages
OCR FSMQ Worked Solutions 4th
PDF
No ratings yet
OCR FSMQ Worked Solutions 4th
113 pages
Edexcel International A Level Maths Pure Mathematics 1 Wma1101 661043a991815405968c3f73 221
PDF
No ratings yet
Edexcel International A Level Maths Pure Mathematics 1 Wma1101 661043a991815405968c3f73 221
8 pages
English Language B - Edexcel IGCSE Resources
PDF
100% (1)
English Language B - Edexcel IGCSE Resources
1 page
As Pure Math Paper 1 Answer
PDF
No ratings yet
As Pure Math Paper 1 Answer
785 pages
Ukuqonda Math Gr9 Learner Book
PDF
No ratings yet
Ukuqonda Math Gr9 Learner Book
297 pages
Complete Download Teaching secondary school mathematics research and practice for the 21st century Geiger PDF All Chapters
PDF
100% (1)
Complete Download Teaching secondary school mathematics research and practice for the 21st century Geiger PDF All Chapters
62 pages
63 Indices Merged
PDF
No ratings yet
63 Indices Merged
276 pages
Edexcel as and a Level Further Mathematics Further Mechanics 2 1nbsped 9781292183329 1292183322 Compress
PDF
No ratings yet
Edexcel as and a Level Further Mathematics Further Mechanics 2 1nbsped 9781292183329 1292183322 Compress
252 pages
Oyarolge M@-Lanleyilolet-Ar-Tave Ma Syan An Ocr Endorsed Textbook
PDF
No ratings yet
Oyarolge M@-Lanleyilolet-Ar-Tave Ma Syan An Ocr Endorsed Textbook
628 pages
O Level Math Matrices Topical
PDF
No ratings yet
O Level Math Matrices Topical
68 pages